TEST PAPER 7 Flashcards

1
Q

1.A middle-aged patient who sustained RTC 6 months ago presents with progressive visual lossand exophthalmos on the right. MRI demonstrated a dilated superior ophthalmic vein withflow voids in the cavernous sinus. What is the likely diagnosis?
A. Buphthalmos
B. Carotid-cavernous fistula
C. Orbital pseudotumour
D. Arteriovenous malformation
E. Dural fistula

A

1.B. Carotid-cavernous fistula
Carotid-cavernous fistula (also described as caroticocavernous fistula) is an abnormalcommunication between the internal carotid artery (ICA) and the veins of the cavernous sinus.
It is mostly due to trauma with laceration of the ICA within the cavernous sinus usually due to a skull base fracture or penetrating trauma. Ultrasound and MRI usually show arterial flow in the cavernous sinus and superior ophthalmic vein.

How well did you know this?
1
Not at all
2
3
4
5
Perfectly
2
Q

2.A preterm neonate on the intensive care unit develops gross abdominal distension andbleeding per rectum. A supine abdominal radiograph is performed demonstrating multipleloops of dilated bowel loops.
Given the likely diagnosis, all of the following would be expected findings on the supine radiograph, except:
A. Mottled gas shadows within the bowel wall
B. Branching gas pattern overlying the liver shadow
C. Foci of calcification projected over the renal angles
D. Generalised lucency overlying the liver shadow
E. Rounded area of lucency within the central abdomen

A

2.C. Foci of calcification projected over the renal angles
The main observations to be made on the plain abdominal radiograph relate primarily to thepresence, amount and distribution of gas, which includes intraluminal gas, intramural gas, portal venous gas and free intraperitoneal gas. From observations of the intraluminal gas, it may sometimes be possible to make inferences regarding the presence of bowel wall thickening, free fluid and focal fluid collections.
Dilatation with loss of the mosaic pattern and the development of rounded or elongated loops is more suggestive that an abnormality is present. On plain abdominal radiographs, intramural gas may be diffuse or localised and appears as linear or rounded radiolucencies. Extensive intramural gas can result in a mosaic pattern or bubbly appearance. Portal venous gas appears as branching, linear, radiolucent vessels that may extend from the region of the main portal vein towards the periphery of both hepatic lobes.
On the supine view, large amounts of gas may give rise to the ‘football’ sign, where the gas outlines the whole of the peritoneal cavity, the undersurface of the diaphragm and the falciform ligament (the lacing of the football). In this view, even smaller amounts of free gas may be detected when both sides of the bowel wall are outlined (Rigler’s sign).

How well did you know this?
1
Not at all
2
3
4
5
Perfectly
3
Q

3.Which of the following is a recognised cause of a ‘bone within bone’ appearance?
A. Renal osteodystrophy
B. Paget’s disease
C. Hyperparathyroidism
D. Melorheostosis
E. Osteopathia striata

A

3.B. Paget’s disease
A ‘bone within bone’ appearance describes the radiographic appearance whereby a bone appearsto have another bone within it, which results from endosteal new bone formation. Recognised causes include Paget’s disease, sickle cell disease, thalassemia, Gaucher’s disease, acromegaly, hypervitaminosis D, scurvy and rickets, among many others. It can also be a normal finding in infants, particularly in the thoracolumbar spine.

How well did you know this?
1
Not at all
2
3
4
5
Perfectly
4
Q

4.A 25-year-old woman with recurrent urinary tract infections and post-void dribbling attendsthe urology clinic. The urologist suspects a urethral diverticulum. What is the mostappropriate first-line test?
A. Micturating cystourethrogram
B. Urodynamics
C. Transvaginal ultrasound
D. Double balloon catheter positive pressure urethrography
F. Pre- and post void magnetic resonance imaging of urethra

A

4.E. Pre- and post-void magnetic resonance imaging of urethra
Female urethral diverticula are thought to be caused by obstruction of Skene’s glands. Rarely,carcinoma (usually adenocarcinoma) or calculi can form within them. The gold standard imaging investigation is double-balloon catheter positive pressure urethrography; however, this is invasive and uses ionising radiation. The best first-line test is a pre- and post void magnetic resonance imaging (MRI) of the urethra, as this avoids ionising radiation and does not involve invasive placement of catheters, and the patient can empty her bladder in private between scans. Positive pressure urethrography is reserved for cases where MRI is equivocal. The remaining options are inappropriate: transvaginal ultrasound may show a lesion but will not clearly demonstrate its relationship to the urethra; micturating cystourethrogram would have a high likelihood of a false negative, requires catheterisation, uses ionising radiation and requires the patient to void in the presence of the radiologist; urodynamics are a functional examination.

How well did you know this?
1
Not at all
2
3
4
5
Perfectly
5
Q

5.A 40-year-old male and intravenous drug user, is on your barium list. The history on the cardsays ‘c/o dysphagia. Exclude pouch!’. During the barium swallow, you notice no oesophagealpouch but there are at least three giant 3- to 4 cm flat ulcers noted within the oesophagus near the gastro-oesophageal junction. The intervening oesophagus appears normal. Which of the following is the most likely diagnosis?
A. Cytomegalovirus oesophagitis
B. Caustic oesophagitis
C. Candida oesophagitis
D. Behcet’s disease
E. Crohn’s disease

A
  1. A. Cytomegalovirus oesophagitis
    Cytomegalovirus oesophagitis almost always occurs exclusively in human immunodeficiency virus (HIV)-positive patients. Barium swallow or endoscopy appearances are of giant ovoid flat ulcers >2 cm near the gastro-oesophageal junction or less commonly, smaller superficial ulcers. Giant ulcers can also be seen in HIV at the time of seroconversion. Caustic ingestion would usually have a different clinical history and tends to produce stricture of the oesophagus. Candida tends to involve the upper oesophagus, has linear plaques and is associated with abnormal motility. Behcet’s disease (a rare immune-mediated systemic vasculitis) produces aphthous ulcers. Crohn’s disease rarely affects the oesophagus and also would produce aphthous ulcers.
How well did you know this?
1
Not at all
2
3
4
5
Perfectly
6
Q
  1. A 16-year-old girl with a history of recurrent bronchitis undergoes chest X ray. The lungs are clear but there is tracheal deviation to the left, with a focal indentation of the right wall of the trachea. Underlying vascular anomaly is suspected, and the patient undergoes a magnetic resonance imaging scan for further evaluation. All of the following will explain the above Chest X-ray appearance, except:
    A. Double aortic arch
    B. Right aortic arch with aberrant left subclavian artery and patent ductus arteriosus
    C. Aberrant left pulmonary artery
    D. heft aortic arch with aberrant right subclavian artery and patent ductus arteriosus
    E. Common origin of innominate and left common carotid artery
A
  1. C. Aberrant left pulmonary artery
    Double aortic arch variants and right aortic arch with aberrant left subclavian artery and patent ductus arteriosus are the two most common types of vascular rings which encircle the mediastinal airways. Both these conditions cause leftward deviation of the trachea and indentation of the right tracheal wall visible on the chest X-ray together with a large posterior oesophageal impression visible on oesophagogram. Left aortic arch with aberrant right subclavian artery is the most common vascular anomaly of the aortic arch, but only in the extremely rare association with patent ductus arteriosus will it cause similar appearances.
    A less common vascular anomaly, which may cause a similar appearance of the trachea but does not cause any oesophageal indentation, is a common origin of innominate and left common carotid artery. Aberrant left pulmonary artery’ causes posterior tracheal indentation and anterior oesophageal impression.
How well did you know this?
1
Not at all
2
3
4
5
Perfectly
7
Q
  1. A 50 year-old man was recently diagnosed with a thyroid cancer following an ultrasound guided FNA of a thyroid lesion. Regarding malignant thyroid nodules, which of die following statements is true?
    A. Punctate calcification is a feature of papillary carcinoma.
    B. Anaplastic carcinoma is associated with MEN syndrome.
    C. Echogenic foci seen in medullary carcinoma are due to calcitonin deposits.
    D. Characteristic lymphadenopathy in medullary carcinoma is hypoechoic to muscle.
    E. Follicular carcinoma can he differentiated from follicular adenoma on US.
A
  1. A. Punctate calcification is a feature of papillary carcinoma.
    Thyroid calcifications may occur in both benign and malignant diseases. Thyroid calcifications can be classified as microcalcification, coarse calcification or peripheral calcification. Microcalcifications are found in 29%-59% of all primary thyroid carcinomas, most commonly in papillary thyroid carcinoma. Their occurrence has been described in follicular and anaplastic
    thyroid carcinomas as well as in benign conditions such as follicular adenoma and Hashimoto’s thyroiditis.
    At US imaging, microcalcifications appear as punctate hyperechoic foci without acoustic shadowing. Coarse calcifications may coexist with microcalcifications in papillary cancers, and they are the most common type of calcification in medullary thyroid carcinomas. Inspissated colloid calcifications in benign thyroid lesions may mimic microcalcifications in thyroid malignancies, but the former can be distinguished from malignant calcifications by the observation of ring-down or reverberation artefact. Peripheral calcification is one of the patterns most commonly seen in a multinodular thyroid but may also be seen in malignancy.
    US features that should arouse suspicion about lymph node metastases include a rounded bulging shape, increased size, replaced fatty hilum, irregular margins, heterogeneous echotexture, calcifications, cystic area and vascularity throughout the lymph node instead of normal central hilar vessels at Doppler imaging.
How well did you know this?
1
Not at all
2
3
4
5
Perfectly
8
Q
  1. A 6-year-old boy presents with a 1-month history of progressive left-sided proptosis. An orbital MRI reveals a large, lobulated, retro-orbital mass without any intracranial or globe invasion. The mass is isointense to muscle on Tl and hyperintense on T2 with uniform enhancement post-contrast. The patient is afebrile, and inflammatory markers are not significantly raised. What is the most likely diagnosis?
    A. Dermoid cyst
    B. Orbital cellulitis with abscess formation
    C. Lymphangioma
    D. Capillary’ haemangioma
    E. Rhabdomyosarcoma
A
  1. E. Rhabdomyosarcoma
    Rhabdomyosarcoma is the most common mesenchymal tumour in children. Rhabdomyosarcoma is an aggressive, rapidly growing tumour and most often manifests with rapidly progressive proptosis or globe displacement. Orbital cellulitis is differential, but patients are afebrile and inflammatory markers are normal. On CT images, orbital rhabdomyosarcoma generally appears as an extraconal, irregular ovoid, well-circumscribed, homogeneous mass that is isoattenuated relative to muscle. Calcification is usually seen only in association with bone destruction. At MRI, they are isointense to muscle or brain with T1-weighted sequences and variably hyperintense to muscle and brain with T2-weighted pulse sequences. They enhance uniformly with contrast.
    Dermoid cyst is the most common orbital mass in children. Imaging features that suggest a dermoid include a cystic appearance, internal fat attenuation or signal intensity (T1 hyperintensity) and internal calcification, all of which are uncommon in rhabdomyosarcoma.
    On images, vascular malformations are often cystic and multiloculated with ill-defined borders. They frequently contain fluid-fluid levels because of haemorrhage into the cysts, whereas fluid fluid levels are quite uncommon in rhabdomyosarcoma. Peripheral enhancement can be seen around cystic area, which is uncommon in rhabdomyosarcomas.
How well did you know this?
1
Not at all
2
3
4
5
Perfectly
9
Q
  1. A 30-year-old woman was involved in a severe road traffic accident and sustained direct high-energy trauma to her pelvis. Among other injuries, she was found to have a fracture on her left sacroiliac joint and left ischiopubic ramus. What type of fracture has she sustained?
    A. Open book
    B. Straddle
    C. Bucket handle
    D. Duverney
    E. Malgaigne
A
  1. E. Malgaigne
    Pelvic fractures can be divided into stable and unstable fractures. The Malgaigne, open book, straddle and bucket-handle fractures are all unstable, as the pelvic ring is interrupted in two places. The Malgaigne fracture is described in this case. The open book fracture implies fracture/ diastasis of both ischiopubic rami and sacroiliac joints, the straddle fracture involves both obturator rings and the bucket-handle fracture refers to an SI joint fracture with a contralateral ischiopubic ramus fracture. Patients with unstable fractures are at significant risk of pelvic organ injury and haemorrhage. Duverney fracture is an isolated fracture of the iliac wing and is a stable fracture.
How well did you know this?
1
Not at all
2
3
4
5
Perfectly
10
Q
  1. A patient undergoes a routine abdominal ultrasound for generalised abdominal pain.
    Unfortunately, the spleen cannot be detected. Which of the following is the least likely cause for this?
    A. Myelofibrosis
    B. Sickle cell anaemia
    C. Polysplenia syndrome
    D. Traumatic fragmentation of the spleen
    E. Wandering spleen
A
  1. A. Myelofibrosis
    Myelofibrosis causes splenomegaly and therefore make splenic detection easier. All of the other options provided are potential causes of a non-visualised spleen. Polysplenia syndrome (also known as bilateral left-sidedness) is, as the name suggests, actually associated with multiple spleens, but these are usually in the wrong place (in addition to a vast array of other intra-abdominal anomalies). A wandering spleen relates to the condition where the spleen is attached to an abnormally long and mobile pedicle, which means that the spleen can be found in places other than in the left upper quadrant
How well did you know this?
1
Not at all
2
3
4
5
Perfectly
11
Q
  1. A 6-year-old boy presents with adrenal insufficiency and developmental delay. Magnetic resonance imaging demonstrates diffuse T2 hyperintensity in the deep white matter, most predominant in the posterior parieto-occipital region and splenium of the corpus callosum. Which of the following is the most likely cause for this finding?
    A. Metachromatic leukodystrophy
    B. Acute disseminated encephalomyelitis
    C. X-linked adrenoleukodystrophy
    D. Alexander disease
    F. Canavan disease
A
  1. C. X-linked adrenoleukodystrophy
    Dysmyelinating diseases, or leukodystrophies, encompass a wide spectrum of inherited neurodegenerative disorders affecting the integrity of myelin in the brain and peripheral nerves. Most of these disorders fall into one of three categories lysosomal storage diseases, peroxisomal disorders and diseases caused by mitochondrial dysfunction - and each leukodystrophy has distinctive clinical, biochemical, pathological and radiological features. X-linked adrenoleukodystrophy is an inherited white matter disorder caused by gene mutation (ALD gene) resulting in abnormal formation of myelin. The childhood cerebral form (CCALD) is the mast common and affects males aged between 4 and 10 years. Hyperpigmentation can occur as a result of adrenal insufficiency. The diagnostic clue is symmetric, peritrigonal white matter abnormality involving the splenium. Alexander disease characteristically involves the frontal white matter preferentially, and Canavan disease causes diffuse white matter abnormality.
How well did you know this?
1
Not at all
2
3
4
5
Perfectly
12
Q
  1. A 26-year-old woman who had an intrauterine contraceptive device (IUCD) coil inserted 6 years ago presents to her general practitioner complaining of right iliac fossa pain, constipation, night sweats and fevers. The practitioner refers her for a transvaginal ultrasound, which shows a right-sided convoluted cobra-shaped structure containing fluid echogenicity and some polypoidal outgrowths from the wall. Adjacent to this is
    a cystic left adnexal mass containing internal echoes. Which of the following is the likely diagnosis?
    A. Actinomycosis
    B. Appendix abscess
    C. Diverticulitis with pericolic abscess
    D. Migrated IUCD causing hydroureter
    E. Salpingitis secondary to tuberculosis
A
  1. A. Actinomycosis
    Pelvic actinomycosis is a rare chronic bacterial infection but is commonly seen in the setting of a long-standing IUCD. It can also be associated with recent surgery. It often causes abdominal pain, low grade fever and an abdominal or pelvic mass/abscess, which can mimic a malignant mass as it can get quite large if left untreated. The earlier it is diagnosed and treated, the less likely the patient will require surgery. The cobra-shaped structure is an infected dilated tube. Endometriosis can give cysts containing low-level echoes but fever would be unusual with pain. Appendix and diverticulitis can cause irritation of the adjacent tube but the clue is the long-standing IUCD.
How well did you know this?
1
Not at all
2
3
4
5
Perfectly
13
Q
  1. A 75-year-old man had a history of dyspnoea associated with haemoptysis and weight loss. Computed tomography (CT) showed a 1.5 cm spiculated mass in the anterior segment of the right upper lobe, 5 cm deep to the pleural surface on a background of widespread emphysematous change. The case was referred for discussion at multidisciplinary team meeting to consider safety of undergoing a CT-guided lung biopsy. In this patient’s case, which of the following statements concerning CT-guided lung biopsy is correct?
    A. The patient carries a 10%-15% risk of developing pneumothorax.
    B. As the lesion is not contiguous with the pleural surface, there is a lower risk of pneumothorax.
    C. The patient carries an increased risk of developing pulmonary haemorrhage post-procedure.
    D. If a pneumothorax were to develop as a complication, he is less likely to require subsequent intercostal drain insertion.
    E. The procedure is relatively contraindicated because pulmonary function tests revealed a forced expiratory volume in 1 second (FEV1) of 45% predicted
A
  1. C. This patient carries an increased risk of developing pulmonary haemorrhage
    post-procedure.
    Studies have identified lesion depth as being the most important risk factor for pulmonary haemorrhage, with an increased risk in lesions deeper than 2 cm. The incidence of pneumothorax is reported to be between 22% and 45%, of which 3.3-15% will require a chest drain. The risk of developing pneumothorax increases significantly if the lesion is not contiguous with the pleural surface. Studies have shown that the presence of chronic obstructive airway disease increases the necessity of chest drain insertion, although this does not necessarily hold for risk of pneumothorax. Although there are no definite absolute contraindications to CT guided lung biopsy, there are, however, several relative contraindications. Patients should not undergo the procedure without adequate prebiopsy assessment or if they plan to fly within 6 weeks of the procedure. The risk is increased by abnormalities of lung function, respiratory failure (including mechanical ventilation), arterial and venous pulmonary hypertension and coagulation abnormalities. The balance of benefit against risk for the procedure should be assessed at a multidisciplinary meeting. Previous contralateral pneumonectomy precludes needle biopsy; however, if the lesion abuts the pleural surface and can be accessed with no needle traversing lung tissue, then the risk of pneumothorax is very low and may not, therefore, be considered as a contraindication. Patients should not undergo needle biopsy without further multidisciplinary team assessment if pulmonary function tests demonstrate an FEV1 of <35%.
How well did you know this?
1
Not at all
2
3
4
5
Perfectly
14
Q
  1. A 58-year-old man, who underwent coronary artery bypass grafting 8 years ago, presents to the cardiology clinic with symptoms of progressive shortness of breath on exertion, associated with increase in abdominal girth and peripheral oedema. Clinical examination elicits raised jugular venous pressure, with bibasal fine inspiratory crackles, shifting dullness and bilateral ankle pitting oedema. Elective ECG-gated spin-echo cardiac MRI demonstrates limited ability of the right ventricle to distend during filling (diastole), assuming a tubular shape, with limited change in cavity size during the end-systolic phase. Pericardial thickening of 6 mm and calcification is evident, with a moderate pericardial effusion and dilated superior vena cava and azygos vein. Which of the following is the most likely diagnosis?
    A. Cardiac tamponade
    B. Restrictive cardiomyopathy
    C. Constrictive pericarditis
    D. Dressier syndrome
    E. Hypertrophic cardiomyopathy
A
  1. C. Constrictive pericarditis
    Constrictive pericarditis is a recognised complication of cardiac surgery, with recent evidence showing an incidence of 0.2%. The most common cause, however, would be idiopathic, thought to result from an occult viral pericarditis, with tuberculosis being the most common cause worldwide. Clinical symptoms attributed to both left- and right-sided heart failure are usually present.
    The hallmarks of pericardial constriction are pericardial thickening, pericardial calcification and abnormal diastolic ventricular function. Other findings associated with raised right-sided pressure such as dilatation of the superior vena cava and azygos vein help support the diagnosis. Constrictive pericarditis can be distinguished from restrictive cardiomyopathy on the basis of pericardial thickness measuring more than 4 mm in the presence of characteristic haemodynamic findings.
    Dressier syndrome typically occurs 3 weeks to several months, not years, after cardiac surgery.
How well did you know this?
1
Not at all
2
3
4
5
Perfectly
15
Q
  1. An 8-month-old boy presents with a right upper quadrant mass. Blood results reveal a raised alpha fetoprotein (AFP). Ultrasound of the abdomen demonstrates a large 7-cm, hypervascular, heterogeneous hyperechoic mass in the liver with a few cystic regions.
    There is no vascular invasion. No renal or suprarenal lesions are present. Which of the following differential diagnoses is most likely?
    A. Hepatoblastoma
    B. Infantile haemangioendothelioma
    C. Hepatic haemangioma
    D. Mesenchymal hamartoma of the liver
    E. Fibrolamcllar hepatocellular carcinoma
A
  1. A. Hepatoblastoma
    Hepatoblastoma is the most common primary hepatic tumour in children. Hepatoblastoma has been associated with several syndromes, including Beckwith-Wiedemann syndrome,
    Gardner syndrome, familial adenomatous polyposis, type 1A glycogen storage disease and trisomy 18.
    Hepatoblastomas are most often hyperechoic relative to adjacent liver on US. A spoke-wheel appearance with areas of alternating echogenicity may be seen at antenatal imaging. CT shows a sharply circumscribed mass that is slightly hypoattenuating relative to the adjacent liver on unenhanced and contrast-enhanced images. Epithelial hepatoblastomas demonstrate a more homogeneous appearance, while mixed tumours are more heterogeneous in attenuation. Speckled or amorphous calcification is seen in more than 50% of lesions. The tumour enhances slightly, but less than adjacent liver.
    At MR imaging, epithelial hepatoblastomas are homogeneously slightly hypointense on T1-weighted images and hyperintense on T2-weighted images relative to adjacent liver parenchyma. Mixed tumours demonstrate more heterogeneous signal intensity characteristics. Fibrotic septa are hypointense on both Tl- and T2-weighted images and enhance after intravenous administration of gadolinium contrast material.
    Infantile haemangioendothelioma (IHE) is a vascular tumour and enhances much more than adjacent liver, while hepatoblastoma typically enhances much less than adjacent liver. Occasionally, the peripheral rim enhancement on arterial phase images seen in hepatoblastoma may suggest IHE, but IHE is distinguished by intense nodular or corrugated peripheral enhancement with centripetal fill-in on delayed phase images. Mesenchymal hamartoma of the liver (MHL) is a benign tumour that manifests in the same age group as hepatoblastoma. It can usually be distinguished from hepatoblastoma by normal serum AFP levels in MHL, predominantly cystic appearance and age at diagnosis >5 years (cf. hepatoblastoma generally diagnosed <5 years).
How well did you know this?
1
Not at all
2
3
4
5
Perfectly
16
Q
  1. What is the purpose of the heel-toe manoeuvre in ultrasound examination of the shoulder?
    A. To decrease the beam angle incidence
    B. To minimise anisotropy
    C. To increase the field of view
    D. To minimise posterior reverberation artefact
    E. To minimise beam width artefact
A
  1. B. To minimise anisotropy
    The purpose of the heel-toe manoeuvre in ultrasound is the same in all situations regardless of the site being imaged. It is to reduce anisotropy artefacts that result when the probe is not perpendicular to the structure being imaged. Similarly, reducing beam angle incidence would increase such artefacts. Reducing probe compression reduces posterior reverberation artefact. Placing the structure of interest within the central region of the probe and placing the focal zone at the region of interest reduce beam width artefact.
How well did you know this?
1
Not at all
2
3
4
5
Perfectly
17
Q
  1. A 23-year-old woman presents with left iliac fossa pain. The uterus and both ovaries are within normal limits. There is an anechoic left adnexal cyst adjacent to the uterus that appears separate from the ovary. You note that the patient has had a previous ultrasound for left adnexal pain, and that a left adnexal cyst with similar dimensions was noted then. Which of the following is the most likely diagnosis?
    A. Theca lutein cyst
    B. Paraovarian cyst
    C. Endometrioma
    D. Adenomyosis
    E. Dermoid cyst
A
  1. B. Paraovarian cyst
    Paraovarian cysts arc responsible for about 10% of adnexal masses. They do not usually change in size. They are susceptible to torsion or bleeding which can cause pain but are usually asymptomatic. They are congenital, occurring from embryonic Wolffian or mesonephric duct remnants within the broad ligament, and are separate from the ovary. Theca lutein cysts are usually bilateral.
    Endometriomas usually contain low-level echoes and there is often more than one deposit; they are also less likely to remain static over time. Adenomyosis is a uterine abnormality that involves abnormal glandular tissue within the myometrium and causes thickening of the myometrium.
    An ovarian dermoid would usually be hyperechoic or contain mixed elements, giving it a heterogeneous appearance.
How well did you know this?
1
Not at all
2
3
4
5
Perfectly
17
Q
  1. A patient undergoes pancreatic transplantation. Which of the following statements is least likely?
    A. The transplanted pancreatic duct is normally dilated.
    B. Indistinct pancreatic margins on ultrasound may indicate graft rejection.
    C. Most patients have a simultaneous renal transplant
    D. The donor pancreas is normally grafted onto the external iliac vessels.
    E. Most patients achieve insulin independence
A
  1. A. The transplanted pancreatic duct is normally dilated.
    Pancreatic transplant is a potentially curative treatment option predominantly for Type 1 diabetes. The pancreas is normally grafted onto the external iliac vessels. In most cases, the procedure is combined with a simultaneous renal transplant. Exocrine pancreatic secretions can be redirected either into the bladder (easier with simultaneous renal transplant) or bowel. There are advantages and disadvantages of both approaches; redirection to the bladder enables close monitoring of secretions but can lead to acidosis. Radiological input is usually required in postoperative monitoring. Ultrasound scanning can reveal peripancreatic collections/pseudocysts, vessel thrombosis and other signs of acute rejection (indistinct pancreatic margins, acoustic inhomogeneity of the pancreas and dilatation of the pancreatic duct).
How well did you know this?
1
Not at all
2
3
4
5
Perfectly
18
Q
  1. A 30-year-old woman presents with bilateral foot drop 2 days post-partum. What finding on MRI would explain this?
    A. Posterior disc protrusion at L3/L4
    B. Bilateral common peroneal nerve entrapment
    C. Bilateral sciatic nerve compression
    D. Paracentral disc protrusion at L3/L4
    E. Spinal canal stenosis at L3/L4
A
  1. B. Bilateral common peroneal nerve entrapment
    Common peroneal neuropathy (CPN) is the most common mononeuropathy in the lower extremity. In most cases, CPN neuropathy occurs in the knee region, whereas neuropathy of the superficial peroneal nerve (SPN) and deep peroneal nerve (DPN) occurs more distally in the leg, ankle or foot.
    The CPN is particularly prone to entrapment because it is fixed in position at the greater sciatic foramen (peroneal division) and around the fibular head. There are two common compression sites of the CPN. The nerve may be compressed as it crosses the fibular neck, owing to its superficial location, or as it travels under the origin of the peroneus longus muscle. Injury to the nerve at these locations may be the result of extrinsic compression, stretch injury or direct trauma.
    Extrinsic compression of the CPN can be the result of external compression by various agents such as short-leg cast, crush injury, surgery, tumour, osteochondroma, synovial cyst, intraneural
    and extraneural ganglia, varicosities, aberrant muscle, prolonged immobilisation (Saturday night palsy), prolonged squatting (strawberry pickers’ palsy) and extended lithotomy position due to childbirth or obstetric surgery will typically produce bilateral CPN entrapment. Diabetic patients are at an increased risk for entrapment of the CPN within the fibrous tunnel underneath the peroneus longus muscle.
    Patients with CPN often present with frequent tripping related to a foot drop. Pain may be present at the site of compression. Sensory disturbances include paraesthesia and anaesthesia along the lateral lower leg and dorsal foot. On physical examination, patients demonstrate foot drop, weak foot extension (anterior tibial muscle), weak foot eversion (peroneus longus and brevis muscles) and loss of sensation in the lower lateral two-thirds of the leg and the dorsum of the foot.
How well did you know this?
1
Not at all
2
3
4
5
Perfectly
19
Q
  1. On an antenatal ultrasound, a foetus is found to have an intracranial anomaly. At birth, the cranial ultrasound reveals a large cystic mass in the posterior fossa communicating with the fourth ventricle with hypoplasia of the cerebellar vermis.
    What other associated abnormality would you not expect to be associated with the underlying condition?
    A. Subependymal calcification
    B. Corpus callosum agenesis
    C. Grey matter heterotopia
    D. Schizencephaly
    E. Occipital encephalocoele
A
  1. A. Subependymal calcification
    Dandy-Walker malformation is the most common posterior fossa malformation. The key neuroimaging features are hypoplasia (or, rarely, agenesis) of the cerebellar vermis (whose inferior portion is typically affected, possibly in combination with its superior portion), which is elevated and upwardly rotated; and dilatation of the cystic-appearing fourth ventricle, which consequently may fill the entire posterior fossa.
    Additional malformations, including dysgenesis or agenesis of the corpus callosum, occipital encephalocele, polymicrogyria and grey matter heterotopia, may be present in 30% 50%. Hydrocephalus is associated in about 90% of patients.
    Subependymal calcification is a feature of tuberous sclerosis.
How well did you know this?
1
Not at all
2
3
4
5
Perfectly
20
Q
  1. A 13-year-old boy presents with symptoms and radiographic evidence of a slipped capital femoral epiphysis (SCFE). It is noted on his radiographs that the physes are generally wide with flaring of the metaphyses. Which of the following is the most likely diagnosis?
    A. Rickets
    B. Hypophosphatasia
    C. Blounts disease
    D. Achondroplasia
    E. Renal osteodystrophy
A
  1. A. Rickets
    Rickets is the paediatric equivalent of osteomalacia. It affects the metaphysis of hones as these are the most metabolically active sites. Common sites of involvement include proximal humerus, proximal tibia and proximal and distal femur. Appearances include widened and irregularly shaped physeal lucencies and metaphyseal flaring. There may be long bone deformation with lower limb bowing. Patients with rickets are at increased risk of Salter-Harris I fractures of the epiphyses that most commonly occur at the proximal femur (SCFE).
    While Blount’s disease is often associated with bow legged-ness in infants and children, it is an abnormality at the knee with an increase in the tibial metaphyseal angle. There arc a few cases in the literature of SCFE in association with Blount’s disease.
    While hypophosphatasia may give similar findings to rickets, its incidence 1:10 0000 is several orders of magnitude less than rickets (1:100-1:1000).
How well did you know this?
1
Not at all
2
3
4
5
Perfectly
21
Q
  1. A 76 year-old woman is having a pelvic magnetic resonance imaging (MRI) scan to assess for a possible hernia. She is noted to have a 6 cm very low intensity lesion within the right ovary on both Tl and T2. Some fluid is also noted within the pelvis. Which of the following is the most likely diagnosis?
    A. Ovarian mucinous cystadenoma
    B. Krukenberg tumour
    C. Ovarian fibroma
    D. Dermoid cyst
    E. Clear cell carcinoma of the ovary
A
  1. C. Ovarian fibroma
    Ovarian fibromas typically arise in postmenopausal women and are usually asymptomatic. Rarely, they can cause pressure-type symptoms if they get large enough, or tort causing pain.
    Ovarian fibromas are classically known to cause Meigs syndrome, which is the combination of ascites, pleural effusion and a benign ovarian tumour. The ascites and pleural effusion often resolve after tumour removal. They are also more common in Gorlin (basal cell naevus) syndrome, where they tend to occur at a younger age and are more likely to be bilateral. Owing to their highly fibrous component, they have a similar intensity to uterine fibroids, being low on IT and T2-weighted sequences. On ultrasound, they are often hypoechoic with attenuation of the ultrasound beam as it passes through the lesion. About 1% can undergo malignant transformation to a fibrosarcoma.
    Krukenberg tumours result from ovarian metastases, classically from the stomach; however, colon, breast, lung, gynaecological tumours, sarcomas and melanoma can also spread to the ovary.

These tumours would less likely be so low intensity on Tl and T2-weighted images and there may be a history of cancer. Ovarian mucinous cystadenomas tend to contain a jelly-like fluid, so are usually cystic on imaging, but can contain solid components. A dermoid cyst usually contains mixed elements including fat and would not usually cause ascites. Clear cell carcinoma is an aggressive ovarian lesion associated with endometriosis and a poor prognosis; these often have cystic and solid components.

How well did you know this?
1
Not at all
2
3
4
5
Perfectly
22
Q
  1. You are reviewing the X-rays of a 44-year-old male patient who has complained of mild breathlessness and a cough but otherwise well. Several chest radiographs performed over an 18 month period demonstrate diffuse ground glass shadowing with several scattered confluent areas of air-space consolidation. The lung changes do not appear
    to have any zonal predilection, and no mediastinal, hilar or cardiac abnormality is evident. HRCT also showed fairly extensive smooth interlobular septal thickening.
    The intervening lung appears normal, and there is sharp demarcation between the abnormal and normal lung parenchyma. Which of the following is most likely given the radiological findings described?
    A. Pulmonary vasculitis
    B. Pulmonary oedema
    C. Primary tuberculosis
    D. Alveolar proteinosis
    E. Extrinsic allergic alveolitis
A
  1. D. Alveolar proteinosis
    Alveolar proteinosis is a rare disorder that is characterised by the abnormal accumulation of proteinaceous material in alveoli, secondary to altered surfactant homeostasis. It affects young to middle-aged adults and is more common in men. There is a strong association with cigarette smoking. Clinical features are variable, with symptoms usually being of gradual onset. Chest radiography typically demonstrates bilateral air-space opacity with either an ill-defined nodular or ground glass pattern. An important discriminator from pulmonary oedema is the presence of perihilar lung changes in the absence of cardiomegaly, pulmonary venous hypertension and pleural effusions. Similarly, while sarcoidosis can mimic many lung conditions, the absence of lymphadenopathy is an important feature to note in alveolar proteinosis. The classic computed tomography finding is known as crazy paving - the description given to the combination of patchy ground-glass opacities with smooth interlobular septal thickening in a geographical distribution.
How well did you know this?
1
Not at all
2
3
4
5
Perfectly
23
Q
  1. An otherwise healthy 44-year-old patient presents acutely unwell with new-onset epigastric pain and is found to have a significantly raised amylase level. Which of the following clinical scenarios is least likely?
    A. Alcoholic patient, recent 48-h binge
    B. Previous bouts of right upper quadrant abdominal pain
    C. Recent flu like symptoms
    D. Computed tomography report describing a 6 cm pseudocyst
    E. Fulminant haemolytic-uraemic syndrome
A
  1. D. Computed tomography report describing a 6 cm pseudocyst
    The clinical scenario presented is that of new-onset acute pancreatitis. Pseudocysts are encapsulated collections of pancreatic fluid found in a peripancreatic position. Classically, these take 4 weeks to develop. The other options provided are all potential causes of acute pancreatitis. Alcoholism and cholelithiasis are the most common causes. Recent viral infection (e.g., mumps, hepatitis, glandular fever), trauma, structural anomalies (pancreas divisum), some drugs (e.g., steroids, azathioprine, diuretics) and multisystem conditions (shock, haemolytic-uraemic syndrome, systemic lupus erythematosus) are all recognised causes of pancreatitis. In a large number of patients, no definite cause is ever identified (idiopathic).
How well did you know this?
1
Not at all
2
3
4
5
Perfectly
24
Q
  1. A 73-year-old woman with weight loss, previous history of endometriosis and a CA-125 of 983 μg/mL, attends for an magnetic resonance imaging (MRI) scan of the pelvis after a cystic mass with nodules was noted in the left adnexa on ultrasound. There is a 6 cm predominately unilocular cystic mass in the left adnexa, which is bright on T1W & T1W fat-saturated images with enhancing solid mural nodules along its wall. It remains high signal on fat-saturated imaging. Which of the following is the most likely diagnosis?
    A. Dysgerminoma
    B. Brenner tumour of the ovary
    C. Endometrioma
    D. Ovarian dermoid
    E. Clear cell carcinoma of the ovary
A
  1. E. Clear cell carcinoma of the ovary
    Clear cell carcinoma and endometrioid carcinomas are commonly found in patients with previous endometriosis. Clear cell carcinoma is an aggressive carcinoma. It is frequently cystic with enhancing mural nodules. The cystic and solid components are high signal on both Tl- and T2-weighted images. Brenner tumour of the ovary is low signal on Tl- and T2-weighted imaging. Endometrioma may have a bright Tl signal but should not have enhancing mural nodules or be associated with weight loss. Dysgerminoma is a solid germ cell tumour of the ovary, typically solid with a fibrous capsule and fibrous septa. Ovarian dermoid should contain fat and would not be associated with weight loss.
How well did you know this?
1
Not at all
2
3
4
5
Perfectly
25
Q
  1. A 32-year old lady presents with acute sudden onset headache to the A&E department. CT shows haemorrhage within the fourth ventricle. Which vessel is most likely to be involved?
    A. Anterior cerebral communicating artery
    B. Anterior cerebral artery
    C. Posterior cerebral artery
    D. Posterior cerebral communicating artery
    E. Posterior inferior cerebellar artery
A
  1. E. Posterior inferior cerebellar artery
    With posterior inferior cerebellar artery (PICA) aneurysms, the rate of intraventricular haemorrhage is high due to reflux of blood. If there is an isolated intraventricular haemorrhage, a peripheral PICA aneurysm. King in or near the fourth ventricle, may be suspected.
How well did you know this?
1
Not at all
2
3
4
5
Perfectly
26
Q
  1. A 4-year-old child undergoes a CXR for suspicion of chest infection. The request mentions that the child has a congenital cardiac anomaly, which is, as yet, untreated. No further information regarding the type of anomaly is provided. The only abnormalities you can detect on the CXR include mild generalised cardiomegaly and increased pulmonary arterial flow. You note from the A&E department notes that the child is not cyanosed. What is the most likely diagnosis?
    A. Ebstein anomaly
    B. Ventricular septal defect
    C. Tetralogy of Fallot
    D. Atrial septal defect
    E. Truncus arteriosus
A
  1. B. Ventricular septal defect
    Congenital cardiac anomalies can be categorised according to whether or not the child is cyanosed. Thereafter, assessment of both heart size and pulmonary arterial flow allows considerable shortening of the differential diagnosis:

In the case provided, the only conditions not associated with cyanosis are ventricular septal defect (VSD) and atrial septal defect (ASD). VSD is the most common congenital heart anomaly.

How well did you know this?
1
Not at all
2
3
4
5
Perfectly
27
Q
  1. An elderly patient presents to the orthopaedic clinic with progressive hip pain. He has a history of a hip replacement performed 5 years ago. A radionuclide bone scan is ordered and demonstrates increased uptake around the proximal aspect of the prosthesis. This finding is sensitive and specific for which of the following conditions?
    A. Prosthetic fracture
    B. Mechanical loosening
    C. Periprosthetic fracture
    D. Particle disease
    E. None of the above
A
  1. E. None of the above
    A cemented component in a total hip joint replacement (THR) will demonstrate increased uptake on a radionuclide bone scan for up to 1-2 years. Following this time, increased uptake is good evidence to support mechanical loosening or infection with a 50%-100% sensitivity. It is, however, not specific and therefore correlation with the patients clinical presentation and plain radiography is required. Aspiration under fluoroscopic guidance may be required to diagnose or exclude infection. An arthrogram can be performed at the same time, which may allow a confident diagnosis of mechanical loosening to be made; however, a negative arthrogram does not exclude this.
    Cementless THR components may have persistently increased radionuclide uptake. This is secondary to bone ingrowth into die prosthesis and needs to be differentiated from pathology.
How well did you know this?
1
Not at all
2
3
4
5
Perfectly
28
Q
  1. A 68-year-old male patient presents with painless jaundice. Abdominal ultrasound reveals both intrahepatic and extrahepatic biliary dilatation. The gallbladder is thin walled and there are no gallstones. No other significant abnormality is detected, but the report mentions that ‘… the pancreas was not visualized due to overlying bowel gas What is the most likely underlying diagnosis?
    A. Pancreas divisum
    B. Pancreatic acinar cell carcinoma
    C. Pancreatic adenocarcinoma
    D. Pancreatic islet cell tumour
    E. Pancreatic pseudocyst
A
  1. C. Pancreatic adenocarcinoma
    Painless jaundice commonly occurs secondary to tumours in the head of pancreas. The most common pancreatic tumour, by far, is adenocarcinoma. All the other tumours described are rare by comparison. Pancreas divisum is an anatomical variant relating to pancreatic duct morphology where rather than having a single pancreatic duct, the patient has two (embryologically, there is a failure of fusion); it has an association with idiopathic, recurrent pancreatitis but not with bile duct obstruction. Pancreatic pseudocyst is an encapsulated collection of pancreatic fluid found either within the pancreas or in a peripancreatic position. It most commonly occurs secondary to acute pancreatitis.
How well did you know this?
1
Not at all
2
3
4
5
Perfectly
29
Q
  1. A 55-year-old man presents with a several-week history of worsening stridor. He has had several chest infections over recent months, with repeated courses of antibiotics providing little relief of symptoms. Mediastinal lymph adenopathy is evident on chest radiograph, with a few intrapulmonary nodules in the periphery of both lungs. Computed tomography confirms the presence of multiple nodules, most numerous in the subpleural region of the right lower lobe. Also of note is a mucosal polypoid lesion in the upper trachea and a further endobronchial lesion just distal to the carina in the right main bronchus. Which condition encompasses these clinical and radiographic findings?
    A. Amyloidosis
    B. Alveolar proteinosis
    C. Pulmonary vasculitis
    D. Histoplasmosis
    E. Hydatid disease
A
  1. A. Amyloidosis
    Amyloidosis is a rare condition, which can affect one specific organ, or present as a systemic illness. The disease is characterised by the deposition of proteinaceous material either in a focal, tumour like lesion or an infiltrative fashion. Within the chest, cardiac involvement is the most commonly seen, with patients presenting with variable symptoms ranging from arrhythmias to cardiac failure. Pulmonary involvement is typically in the setting of multiple previous chest infections. Computed tomography often demonstrates ‘tree-in-bud’ opacity, usually in a peripheral location, at sites of previous pneumonia. Within the tracheo bronchial tree, mass-like lesions arc seen arising from the internal wall, often significantly compromising the airway lumen.
    While the other conditions listed could result in scattered intrapulmonary nodules, the additional endobronchial abnormality makes amyloidosis more likely.
How well did you know this?
1
Not at all
2
3
4
5
Perfectly
30
Q
  1. A neonate is noted to be markedly cyanosed, worsening when she cries. A CXR performed on day 1 is normal. Which of the following types of congenital cardiac anomaly is most likely?
    A. Patent ductus arteriosus
    B. Ebstein anomaly
    C. Coarctation of the aorta
    D. Tricuspid atresia
    E. Tetralogy of Fallot
A
  1. D. Tricuspid atresia
    Congenital cardiac anomalies can be categorised according to whether or not the child is cyanosed. Thereafter, assessment of both heart size and pulmonary arterial flow allows considerable shortening of the differential diagnosis:

In the case provided, the child is cyanosed; this excludes patent ductus arteriosus and coarctation as the cause. Ebstein anomaly classically produces a grossly enlarged heart. While tetralogy of Fallot is the most common congenital cardiac anomaly to cause cyanosis, it has characteristic CXR features; upturned cardiac apex and deficient main pulmonary artery creates a ‘boot-shaped’ heart. In addition, there is usually decreased pulmonary vascularity associated with Fallot’s tetralogy.
Tricuspid atresia is the third most common congenital cardiac cause of cyanosis after tetralogy of Fallot and transposition of the great vessels.

How well did you know this?
1
Not at all
2
3
4
5
Perfectly
31
Q
  1. Pick’s disease affects which of the following?
    A. Fronto-parietal lobe
    B. Temporo-parietal
    C. Temporo-frontal
    D. Parieto-occipital
    E. Frontal
A
  1. C. Temporo-frontal
    Pick’s disease is a neurodegenerative disease, and one of the tauopathies (group of neurodegenerative diseases characterised by abnormal metabolism of tau proteins leading to intracellular accumulation and formation of neurofibrillary tangles, similar to Alzheimer’s disease) characterised by the accumulation of the Pick bodies. It is sometimes used synonymously with fonto-temporal lobar degeneration (FTLD), although strictly it is incorrect since all causes of FTLD isn’t pathologically Pick’s disease.
    The primary imaging abnormality is that of cortical atrophy of the frontal and temporal lobes. These changes can be markedly asymmetric and affect one region much more than another. Volume of the head of the caudate nucleus may also be reduced. Differentials include Alzheimer’s disease and corticobasal degeneration where parietal lobe involvement is more pronounced.
How well did you know this?
1
Not at all
2
3
4
5
Perfectly
32
Q
  1. A 36-year-old woman is being investigated for possible renal stones with CT KUB.
    An incidental “polka-dot” appearance to the T12 vertebral body is noted on the axial CT images. MRI at the same level shows characteristic high signal in the vertebral body on Tl and T2W images. All of the following statements regarding this entity are true, except
    A. They may extend to involve the posterior element.
    B. They are commonly multiple.
    C. Compressive lesions are common in the lumbar spine.
    D. Low T1W lesions are more likely to be active.
    E. Most are asymptomatic.
A
  1. C. Compressive lesions are common in the lumbar spine
    Vertebral haemangiomas are a hamartomatous lesion. Spinal haemangiomas arc common and frequently multiple. The prevalence of haemangiomas seems to increase with age and is greatest after middle age, with a slight female predilection. Most haemangiomas are seen in the thoracic and lumbar spine. They are usually confined to the vertebral body, although they may occasionally extend into the posterior elements. Most spinal haemangiomas are asymptomatic. Occasionally, vertebral haemangiomas may increase in size and compress the spinal cord and nerve roots. Compressive vertebral haemangiomas can occur in patients of any age, with a peak prevalence in young adults and preferentially occur in the thoracic spine.
    CT shows the pattern as multiple dots (polka-dot appearance). At scintigraphy, the appearance of osseous haemangiomas ranges from photopenia to a moderate increase in radiotracer uptake. The presence of high signal intensity on T1- and T2-weighted MR images is related to the amount of adipocytes or vessels and interstitial oedema, respectively.
    Fatty vertebral haemangiomas (high on T1-weighted MR) may represent inactive forms of this lesion, whereas low signal intensity at T1-weighted MR imaging may indicate a more active lesion with the potential to compress the spinal cord.
How well did you know this?
1
Not at all
2
3
4
5
Perfectly
33
Q
  1. An alcoholic patient is referred for an ultrasound from the A&E department of your hospital. The request states that the patient has deranged liver function tests and raises the possibility of underlying liver cirrhosis. Which of the following findings would not help you to confirm this diagnosis?
    A. Caudate lobe hypertrophy
    B. Increased echogenicity of the liver parenchyma
    C. Coarse echotexture to the liver
    I). Decreased resistive index in hepatic artery
    E. Hepatofugal flow within the portal vein
A
  1. D. Decreased resistive index in hepatic artery
    There are many features of liver cirrhosis identifiable with ultrasound. Most commonly, these include an echo bright, irregular, shrunken liver (late stage) with a coarse echotexture and hypertrophy of the caudate lobe (earlier in the disease the entire liver can hypertrophy). Isoechoic regenerative nodules may be apparent. If measured, there should be an increase in the resistive index (RI) of the hepatic artery. Reversal of portal venous flow (hepatofugal), portalisation of the hepatic veins, ascites and splenomegaly are extrahepatic signs that are commonly identified. Hepatocellular carcinoma is a major complication of cirrhosis that should always be borne in mind when examining these patients.
How well did you know this?
1
Not at all
2
3
4
5
Perfectly
34
Q
  1. A 53-year-old man is assessed by the receiving surgeon, having presented with severe chest pain, vomiting and sepsis. Mediastinal emphysema is evident on chest radiograph and computed tomography confirms oesophageal perforation. Which of the following statements is incorrect with regard to this condition?
    A. Plain chest radiography is normal in approximately 10% of cases.
    B. Iatrogenic injury is the most common single cause of oesophageal perforation.
    C. Upper oesophageal perforations typically result in right-sided pleural collection.
    I). In blunt chest trauma, the perforation usually occurs in the lower third of the oesophagus.
    E. Water-soluble contrast agents should be used for fluoroscopic assessment.
A
  1. D. In blunt chest trauma, the perforation usually occurs in the lower third of the
    oesophagus.
    Oesophageal perforation is a surgical emergency that carries a high mortality. Oesophageal instrumentation, biopsy, balloon dilatation and attempted intubation (iatrogenic) are the most common causes of rupture and typically result in lower oesophageal injury’. Spontaneous rupture, also known as Boerhaave syndrome, arises when intraluminal pressure is elevated during sudden and severe vomiting episodes. While oesophageal rupture in blunt chest trauma is rare, this typically results in upper oesophageal injury and resultant right-sided hydrothorax.

Chest radiography signs include mediastinal and neck emphysema, mediastinal widening and hydrothorax. Barium can be used in the assessment of patients with possible oesophageal perforation, but only after water-soluble contrast swallow has not shown an abnormality.

How well did you know this?
1
Not at all
2
3
4
5
Perfectly
35
Q
  1. A normally well 8-year-old child presents to the A&E department short of breath and pyrexial complaining of joint pain. Chest X-ray shows an enlarged heart with upper lobe venous blood diversion and small bilateral pleural effusions. No focal collapse/consolidation is evident, but there are patchy interstitial infiltrates in a perihilar distribution. Further questioning reveals that the child recently had a sore throat. What is the most likely underlying diagnosis?
    A. Toxic synovitis
    B. Juvenile rheumatoid arthritis
    C. Reiter’s disease
    D. Rheumatic fever
    E. Septicaemia
A
  1. D. Rheumatic fever
    The imaging features described are those of acute congestive heart failure. In the context of joint pain following a recent sore throat and with no previous cardiac history, rheumatic fever should be considered.
    Rheumatic fever commonly causes a pancarditis with valve insufficiency that can lead to acute congestive heart failure in severe cases. This follows a streptococcal pharyngitis. Chronically, rheumatic heart disease can cause valve stenosis with varying degrees of regurgitation, arrhythmias and ventricular dysfunction. Rheumatic fever as a child is the most common cause of valvular heart disease in adults in the western world. The mitral valve is the most commonly affected. Currently, the disease rarely affects children in the West, but the condition remains common in the developing world.
How well did you know this?
1
Not at all
2
3
4
5
Perfectly
36
Q
  1. All the following cause basal ganglia calcification except?
    A. Pseudohypoparathyroidism
    B. Lead poisoning
    C. Ageing
    D. Hypothyroidism
    E. Wilson’s disease
A
  1. E. Wilson’s disease Basal ganglia calcification:
    Common causes: Hypoparathyroidism, pseudohypoparathyroidism, idiopathic, normal variant, physiologic with ageing.
    Uncommon causes: AIDS encephalopathy, atherosclerosis, birth anoxia, hypoxia, carbonic anhydrase II deficiency, carbon monoxide intoxication, Fahr disease (ferrocalcinosis), familial idiopathic symmetrical basal ganglia calcification and microcephaly, Hallervorden-Spatz disease, haemorrhage, hyperparathyroidism, hypothyroidism, cretinism, Kearns Sayre syndrome, lead encephalopathy, lipoid proteinosis, MELAS syndrome, methotrexate therapy for childhood leukaemia, oculodento-osseous dysplasia, parasitic disease (e.g., toxoplasmosis, cysticercosis), parkinsonism, phenylketonuria variants, pseudopseudohypoparathyroidism, radiation therapy, Down’s syndrome, tuberous sclerosis, viral encephalitis.
How well did you know this?
1
Not at all
2
3
4
5
Perfectly
37
Q
  1. An 82-year-old woman presents with postmenopausal bleeding. Transvaginal ultrasound shows a thickened endometrium and a left adnexal lobulated multicystic lesion, which has some solid elements. Some of the cysts contain fluid-fluid levels and have thick septa. Which of the following is the most likely diagnosis?
    A. Endometriosis
    B. Granulosa cell tumour
    C. Endometrial hyperplasia and haemorrhagic corpus luteal cyst
    D. Serous cystadenocarcinoma
    E. Fibrothecoma
A
  1. B. Granulosa cell tumour
    Many ovarian tumours produce oestrogen, the most common being granulosa cell tumour. This results in postmenopausal bleeding, irregular bleeding in premenopausal women and precocious puberty in prepubertal girls. Breast tenderness is another sign, particularly in postmenopausal women. The fluid levels are caused by haemorrhage into the tumour cysts. Endometriosis tends to improve in postmenopausal women as there is no longer oestrogen being produced; it should not cause a thickened endometrium. Endometrial hyperplasia and haemorrhagic corpus luteal cyst are related to ovulation, so do not occur in postmenopausal women. Serous cystadenocarcinomas may produce adnexal masses but do not produce oestrogen, so do not cause endometrial thickening. Fibrothecomas can produce oestrogen and cause endometrial thickening but tend to be solid hypoechoic lesions.
How well did you know this?
1
Not at all
2
3
4
5
Perfectly
38
Q
  1. A 33-year-old woman with progressive increase in back pain and perineal pain was sent for further evaluation to the spinal surgeons. Axial and coronal reformatted O’ image showed a well-defined lytic lesion of the right upper part of the sacrum with extension through the right sacroiliac joint and absence of a sclerotic rim. All the following arc features of this lesion except?
    A. Most commonly affected bone is sacrum.
    B. Extension into intervertebral disc helps to differentiate GCT from ABC.
    C. Septa show intense enhancement post-contrast injection.
    I). Doughnut sign on scintigraphy usually suggests an alternate diagnosis.
    E. Fluid-fluid level is not specific for this lesion
A
  1. C. Septa show intense enhancement post-contrast injection.
    Giant cell tumour of the spine occurs in the 2nd-4th decades of life, more frequently in females. Sacrum is affected in 90% of such cases. It is usually located in the upper sacrum and sacral wing. Extension to the iliac wing through the sacroiliac joint is possible. Lumbar, thoracic and cervical spine may be affected. It usually predominates in the vertebral body, with frequent involvement of the posterior arch. Extraosseous involvement of the soft tissues is common. Intervertebral disk invasion and extension into an adjacent vertebra are possible.
    In radiography, they appear as lytic lesion with cortical expansion. CT demonstrates lack of a sclerotic rim. Bone scintigraphy shows increased radiotracer uptake. The tumour usually has low-to- intermediate signal intensity on T1-weighted MR images. Areas of high signal intensity suggest haemorrhage. More specifically, they have low to intermediate signal intensity on T2-weighted images, caused by haemosiderin and high collagen content. Enhancement reflects its vascular supply. Cystic areas, haemorrhage, fluid-fluid levels and a peripheral low-signal-intensity pseudocapsule may also be seen.
    ABC occurs between 5 and 20 years but can manifest at any age. There may be a slight female predilection. Thoracic spine is most commonly affected. Spinal involvement is typically in the posterior elements, although extension into the vertebral body is common. Spinal ABC may extend into the adjacent vertebrae or intervertebral disk, ribs and the paravertebral soft tissue. CT and MR imaging typically show a well-defined lesion with internal septation. Fluid-fluid level is better seen on MRI. The predominant bone scintigraphic pattern is moderate to intense radiotracer accumulation at the periphery with little activity at its centre (‘doughnut sign’). Post-contrast images show intense enhancement of the septa. Solid components suggest secondary ABC.
How well did you know this?
1
Not at all
2
3
4
5
Perfectly
39
Q
  1. A 38-year-old woman is undergoing investigations for infertility. She is otherwise asymptomatic. On hysterosalpingography, there is a large filling defect within the uterine fundus with a linear defect that extends into the filling defect. She subsequently has an MRI scan, which demonstrates a myometrial mass with indistinct margins, which abuts the junctional zone and has lower signal on T2 when compared with the adjacent myometrium. There are a few focal high T2 signal intensity areas within, some which
    appear more linear. It demonstrates less enhancement than its adjacent myometrium.
    Which of the following is the most likely diagnosis?
    A. Leiomyosarcoma
    B. Leiomyoma
    C. Endometrial carcinoma
    D. Adenomyosis
    E. Hydatidiform mole
A
  1. D. Adenomyosis
    Adenomyosis can be focal or diffuse, the latter being the more common. The aetiology is unclear, but prominent endometrial glands extend into the myometrium with adjacent smooth muscle hyperplasia. Leiomyoma tends to be sharply demarcated as opposed to ill defined and tends to have low-intensity Tl and T2 signals, although multiple signal characteristics arc seen. It would be unusual for leiomyomas to have linear bands extending from the endometrium; this is more commonly seen in adenomyosis. Leiomyoma and leiomyosarcomas cannot be differentiated accurately on imaging; secondary features of lymphadenopathy or metastasis can help raise the suspicion of leiomyosarcoma. Endometrial carcinoma usually presents in an older age group and tends to present with symptoms of irregular bleeding, which is why it is often diagnosed at an early stage. There would likely be endometrial thickening also. Hydatidiform moles would usually be identified clinically, particularly in a patient being investigated for infertility, as she would have raised beta-human chorionic gonadotrophin levels.
How well did you know this?
1
Not at all
2
3
4
5
Perfectly
40
Q
  1. Before being referred for a CT colonography, a patient asks to speak with a radiologist to clear up a few points regarding colon cancer; Of the following statements, which is most correct?
    A. Smaller polyps are more likely to be malignant than larger ones.
    B. Most colonic polyps are malignant.
    C. Gardener syndrome carries an increased risk of cancer but Peutz-Jeghers syndrome does not.
    D. Most colon cancers start as an adenomatous polyp.
    E. Inflammatory bowel disease carries no increased risk of cancer.
A
  1. D. Most colon cancers start as an adenomatous polyp.
    The vast majority of colon cancers begin as an adenomatous polyp (>90%), although the vast majority of polyps are benign (>90%). With increasing size comes an increased risk of malignant change. In addition, of the three recognised pathological subtypes (tubular, tubulovillous and villous), villous adenomas carry the greatest risk of malignant transformation. There are a large number of other risk factors for developing colon cancer but among them are the hereditary’ polyposis syndromes (Peutz-Jeghers syndrome is less commonly’ related as the polyps are more likely in the small bowel) and the inflammatory bowel diseases ‘Crohn’s disease and ulcerative colitis’.
How well did you know this?
1
Not at all
2
3
4
5
Perfectly
41
Q
  1. A 75-year old man presents with abdominal pain after eating and mesenteric angina is suspected. You decide to perform a celiac axis angiogram using a mechanical contrast pump. What is an appropriate volume and flow rate of contrast to use?
    A. 32 mL @ 8 mL/s
    B. 20 mL @ 20 mL/s
    C. 40 mL @ 20 mL/s
    D. 30 mL @ 20 mL/s
    E. 20 mL @ 6 mL/s
A
  1. A. 32 mL @ 8 mL/s
    The aim is to closely replicate the natural blood flow in the vessel. A hand-injected test run is essential before a pump run to highlight the anatomy and confirm that the catheter tip is not in a subintimal position. An 8/32 injection delivers 8 mL of contrast per second to a total of 32 mL and lasts 4 s. The patients are usually asked to keep as still as possible and hold their breath to minimise artefact caused by motion.
How well did you know this?
1
Not at all
2
3
4
5
Perfectly
42
Q
  1. A 62-year old postmenopausal woman has a magnetic resonance imaging (MRI) scan to look for a hernia. In the right ovary is a 1 cm sharply demarcated low T1 and T2 signal solid mass with adjacent calcification. There is also a multilocular cystic lesion within the same ovary, containing multiple thin-walled septa. Which of the following is the most likely cause of the well-demarcated low-signal lesion?
    A. Ovarian fibroma
    B. Ovarian Brenner tumour
    C. Ovarian dermoid cyst
    D. Endometrioma
    E. Corpus luteal cyst
A
  1. B. Ovarian Brenner tumour
    Brenner tumours are uncommon tumours that are almost always benign. These tumours have a large fibrous component and therefore have a similar appearance to an ovarian fibroma (low signal on Tl- and T2-weighted imaging) on both ultrasound and MRI. Brenner tumours are commonly found with an adjacent epithelial tumour of the same ovary (usually mucinous cystadenoma); hence, in this case, the diagnosis of Brenner tumour is more likely than ovarian fibroma. An ovarian dermoid would usually contain fat but would also be in the differential diagnosis. However, the patient is postmenopausal and so endometrioma and corpus luteal cyst are unlikely.
How well did you know this?
1
Not at all
2
3
4
5
Perfectly
43
Q
  1. Which of the following is specific for osteomalacia?
    A. Brown tumour
    B. Looser zones
    C. Cloaca
    D. Cyclops lesion
    E. Wimberger’s sign
A
  1. B. Looser zones
    Looser zones are a highly specific radiographic feature of osteomalacia. They are sometimes referred to as pseudofractures and are focal linear areas of undermineralised osteoid at sites of mechanical loading. They are often seen bilaterally as linear lucencies that run perpendicular to the bone cortex and do not involve the whole bone width. Common locations include the medial cortex of the femoral necks, the inferior scapula and the ribs.
    These need to be differentiated from bisphosphonate fractures, which start at the lateral cortex of the proximal femur, are often bilateral and are termed as atypical fractures in patients on bisphosphonate therapy.
    Brown tumours are lucent lesions seen in hyperparathyroidism. Cloacas are seen in established osteomyelitis. Wimberger’s sign refers to localised bilateral metaphyseal destruction of the medial proximal tibias. It is a pathognomonic sign for congenital syphilis. Wimberger’s ring sign (sometimes also called just Wimberger’s sign to create confusion) is a radiographic sign seen in scurvy, showing thin pencil-like sclerosis along the epiphyseal margins. The cyclops lesion is an intra-articular mass of fibrotic tissue that may be seen post-anterior cruciate ligament (ACL) graft repair.
How well did you know this?
1
Not at all
2
3
4
5
Perfectly
44
Q
  1. AP radiograph of the leg of a 3-year-old girl shows periosteal reaction in the mid-fibula. Exuberant periosteal reaction was worrisome for malignancy, and MRI was performed. Coronal STIR images showed a low signal intensity fracture line surrounded by extensive soft tissue and bone marrow oedema. No mass was depicted. Other sites commonly affected in toddlers fracture include all the following, except:
    A. Tibia
    B. Humerus
    C. Talus
    D. Calcaneum
    E. Cuboid
A
  1. B. Humerus
    Stress fractures of the lower extremities associated with the onset of ambulation are called toddler’s fractures. These fractures typically occur in children between 9 months and 3 years of age, manifest with a refusal to bear weight and arc not preceded by a recognised acute traumatic event.
    A similar injury can result from subtle torsional forces, such as those occurring in a toddler who stumbles and falls on a positioned foot.
    ‘The typical toddler’s fracture is a non-displaced oblique fracture of the distal portion of the tibia. Although other locations are less common, weight-bearing can also account for toddler’s fractures of the fibula, posterior aspect of the calcaneus, the base of the cuboid and the talus
How well did you know this?
1
Not at all
2
3
4
5
Perfectly
45
Q
  1. A 63-year old man presents with progressive vertical gaze abnormality and cognitive symptoms. MRI of the brain demonstrates volume loss of the mid brain, which was described by the reporting radiologist as Hummingbird sign. What is the diagnosis?
    A. Multisystem atrophy
    B. Progressive supranuclear palsy
    C. Parkinson’s disease
    D. Shy-Drager syndrome
    E. Amyotrophic lateral sclerosis
A
  1. B. Progressive supranuclear palsy
    Conventional MRI is usually not helpful in the diagnosis of early Parkinson’s disease because it most often yields normal findings. In advanced disease, abnormalities of the substantia nigra, including volume loss, decreased T2 signal reflecting iron deposition and blurring of the margins, can be seen. However, the primary role of MRI is to exclude structural abnormalities that potentially mimic Parkinson’s disease (e.g., NPH - normal pressure hydrocephalous, intracranial mass and bilateral subdural haematomas).
    FDG PET images are most often normal and show preserved metabolism in the putamen and globus pallidus. This is a defining feature of Parkinson’s disease and allows differentiation from both PSP (progressive supranuclear palsy) and MSA (multisystem atrophy, Shy-Drager syndrome), which commonly demonstrate reduced basal ganglia FDG activity.
    In patients with MSA-Parkinson’s type, abnormalities are confined to the putamen and include atrophy, symmetric hypointensity on T2 and T2*-weighted images and ‘slitlike’ marginal T2 hyperintensity. Putaminal atrophy appears to help discriminate MSA from Parkinson’s disease, whereas T2 hypointensity is a non specific sign that can be seen in PSP, Wilson’s disease, neurodegeneration with brain iron accumulation and other acquired conditions.
    Patients with PSP exhibit atrophy of the midbrain and tegmentum, manifesting as third ventricular dilatation, reduced midbrain AP diameter or flattening of the superior midbrain. Reduced midbrain AP diameter at the level of the superior colliculi on axial images gives rise to the Mickey Mouse sign. Midbrain atrophy with relative preservation of pons produces the hummingbird sign or penguin sign. Additional findings include superior cerebellar peduncle atrophy and increased FLAIR signal, both of which have reasonably high sensitivity and specificity in distinguishing PSP from Parkinson’s disease and MSA.
How well did you know this?
1
Not at all
2
3
4
5
Perfectly
46
Q
  1. A patient is diagnosed with Helicobacter pylori infection. Which of the following findings would you not expect?
    A. Gastric ulcer
    B. Duodenal ulcer
    C. Linitis plastica
    D. Polypoid gastritis
    E. Thickened gastric folds
A
  1. C. Linitis plastica
    Helicobacter pylori infects the gastric mucosa. In most cases, it is asymptomatic but can be associated with epigastric pain and dyspepsia. Imaging can demonstrate the sequelae of gastritis (thickened folds, polypoidal changes and enlarged areae gastricae), but most commonly patients have either a gastric or duodenal ulcer (or both!). Helicobacter pylori is the most common cause of both gastric and duodenal ulcer diseases.
    Linitis plastica (also known as leather bottle stomach) is a form of gastric stenosis that leads to narrowing of the stomach and which occurs secondarily to a number of conditions but is most frequently seen secondary to malignancy.
How well did you know this?
1
Not at all
2
3
4
5
Perfectly
47
Q
  1. You performed a catheter angiogram and angioplasty for right popliteal stenosis on a patient in the vascular ward and are there to review him the following day. The nurse asks you to prescribe him some analgesia because, overnight, his limb has become painful and pale with loss of power and intolerable tingling. Which of the following is your next step?
    A. Review the puncture site (you find no significant haematoma) and prescribe analgesics
    B. Arrange for the patient to be taken back to the suite for further angiography
    C. You immediately page the vascular surgeon on call
    D. Organise urgent CTPA to exclude pulmonary embolism
    E. You start the patient on low molecular weight heparin and organise US leg veins
A
  1. C. You immediately page the vascular surgeon on call
    You have just heard the dreaded symptoms and signs of compartment syndrome. The patient needs urgent surgical fasciotomy to prevent muscle necrosis. Any salvage treatment can be planned after that.
    Muscle oedema will raise the intracompartmental pressure in the calf fascial compartments. If it is not released promptly, it will lead to muscle necrosis. The patient must have suffered an embolus during the angioplasty procedure. This is a medical emergency and needs urgent treatment.
    The team can consider revascularisation options after this is done.
How well did you know this?
1
Not at all
2
3
4
5
Perfectly
48
Q
  1. A 72-year old poorly controlled diabetic gentleman with multiple foot ulcers undergoes an MRI examination to exclude osteomyelitis. All of the following are MR features of osteomyelitis of bones of the forefoot, except
    A. Low signal intensity in infected bone marrow on T1W images
    B. Contrast enhancement of bone marrow
    C. Decreased signal intensity of bone marrow on STIR images
    D. Increased signal intensity of bone marrow on T2W images
    E. Contrast enhancement of adjacent soft tissues
A
  1. C. Decreased signal intensity of bone marrow on STIR images
    Osteomyelitis is the infection of bone of which the primary modes of pathogenesis are through haematogenous spread, direct spread through trauma or iatrogenic source and extension from adjacent soft-tissue infection. MR is highly sensitive in the diagnosis of osteomyelitis of which the loss of the normal T1-weighted bone marrow fat signal is the most sensitive finding. Other additional findings include increased T2-weighted and STIR signals from the oedema and inflammation. Post-contrast studies may reveal a rim enhancing intraosseous abscess, which would warrant surgical intervention.
49
Q
  1. A malformation is defined as a congenital morphologic anomaly of a single organ or body part due to an alteration of the primary developmental program caused by a genetic defect. All of the following are examples of posterior fossa cystic malformation, except
    A. Blake’s pouch cyst
    B. Dandy Walker malformation
    C. Arachnoid cyst
    D. Rhombencephalosynapsis
    E. Giant cisterna magna
A
  1. D. Rhombencephalosynapsis
    Dandy Walker malformation is characterised by hypoplasia or agenesis of cerebellar vermis and cystic dilatation of the 4th ventricle, which can result in enlargement of the posterior fossa.
    Blake’s pouch cyst is a result of absence of communication between the 4th ventricle and the subarachnoid space leading to tetraventricular hydrocephalus. The cerebellum has a normal size and shape. Typical neuroimaging findings include the presence of a cyst in a retro/infracerebellar location, which is essentially a diverticulum of the enlarged 4th ventricle.
    Giant cisterna magna is an enlarged cisterna magna (>10 mm on mid-sagittal images) with an intact vermis, normal 4th ventricle and, in some patients, an enlarged posterior fossa. Consistent presence of hydrocephalus allows the differentiation of Blake’s pouch cyst from mega cisterna magna.
    Arachnoid cysts are well-defined CSF density extra-axial lesions in the posterior fossa, which do not communicate with the 4th ventricle or the subarachnoid space. They may enlarge during infancy and produce mass effect on the cerebellum and vermis, which may cause a secondary obstruction of the ventricular system, hydrocephalus and/or remodelling or thinning of the overlying occipital bone.
    Isolated hypoplasia of the inferior vermis with normal 4th ventricle has been described variably in literature as Dandy-Walker variant.
    Rhombencephalosynapsis is characterised by the absence of the vermis and continuity of the cerebellar hemispheres, dentate nuclei and superior cerebellar peduncles, which creates a horseshoe shaped arch across the midline, resulting in a keyhole-shaped fourth ventricle. It is a key feature of Gόmez-Lόpez-Hernandez syndrome (parietal alopecia, trigeminal anaesthesia and craniofacial dysmorphic signs) and may be seen with VACTERL.
    Posterior coronal T2-weighted images show the horizontal folial pattern.
50
Q
  1. Which of the following combination of MRI signal is characteristic for cerebral abscess?
A
  1. C. High Low
    The typical appearance of a brain abscess at conventional MRI is that of a ring enhancing lesion, with high signal intensity on T2-weighted images and low or intermediate signal intensity on T1-weighted images. A mature abscess has a low signal-intensity capsule on T2 weighted images.
    However, the presence of a ring-enhancing lesion in the brain is not diagnostic of abscess and must be distinguished from a necrotic neoplasm and other cystic lesions. Data from recent studies suggest that DWI is more sensitive than conventional MRI in distinguishing brain abscesses and cystic tumours. Pus in brain abscesses is strongly hyperintense on trace DWI and has a reduced ADC. On the contrary, most necrotic or cystic brain tumours have intermediate signal intensity on DWI and elevated ADC values
51
Q
  1. A 37-year-old woman, with a previous history of cervical carcinoma and radiotherapy, presents with abdominal pain. On magnetic resonance imaging, there is a large cystic midline structure, which has high Tl signal and some internal debris, extending anteriorly and superiorly from the cervix. Coincidentally, in the cervical region, there is high/intermediate T2 signal surrounding the endocervical canal and disruption of the low-intensity cervical stromal ring. Which of the following is the most likely diagnosis?
    A. Cervical carcinoma recurrence causing haematometra
    B. Cervical carcinoma with invasion of the uterus
    C. Radiotherapy-induced cervical stenosis causing haematometra
    D. Cervical carcinoma recurrence with metastasis to ovary
    E. Imperforate hymen
A
  1. A. Cervical carcinoma recurrence causing haematometra
    Cervical carcinoma tends to be of higher signal on T2 than the surrounding cervical stroma and, in the above case, there is also disruption of the cervical stromal fibrous ring, increasing the likelihood of microscopic parametrial invasion. In this case, the cervical tumour is causing a malignant cervical stenosis and haematometra (high Tl signal is consistent with haemorrhage; tumour is usually intermediate signal/hypointense, similar to myometrium on T1 imaging).
    After radiotherapy to the cervix, cervical stenosis with haematometra can occur; however, post radiotherapy, the cervix becomes low signal because of fibrosis. Ovaries usually lie to either side of the midline, and it would be unusual for a cervical carcinoma metastasis of the ovary to have a high T1 signal. Imperforate hymen is congenital and is normally picked up either neonatally or at the time of menarche, if the latter commonly presents with haematometrocolpos; it would be inconsistent with a cervical cancer history, as this tumour is not seen in patients who are virgo intacta.
52
Q
  1. Which of the following statements is most accurate regarding the use of endoscopic ultrasound?
    A. It is superior to cross-sectional imaging for local staging of oesophageal cancer.
    B. It is superior to cross-sectional imaging in the assessment of liver metastases.
    C. It is less effective than cross-sectional imaging for local staging of rectal cancer.
    D. It is less effective than traditional endoscopy for assessing submucosal masses.
    E. It is not useful in the assessment of a primary pancreatic mass
A
  1. A. It is superior to cross-sectional imaging for local staging of oesophageal cancer. Endoscopic ultrasound (EUS) involves the addition of an ultrasound probe in the endoscopy
    apparatus. This has a number of clinical uses, which are currently being developed. Evidence already exists that EUS is superior to cross-sectional imaging in the local staging (for assessment of both the primary tumour and local nodes) of oesophageal, pancreatic and rectal cancer.
    By visualising the wall layers of the gastrointestinal tract, it is the modality of choice for assessment of submucosal lesions. EUS can be combined with interventional techniques such as drainage or fine needle aspiration cytology. It is useful for differentiating inflammatory and neoplastic pancreatic masses. Cross-sectional imaging remains superior in the assessment of liver metastases.
53
Q
  1. You are called to perform a catheter angiogram on a 26 year-old biker who was involved in a road traffic accident. You are told that he has suffered a dislocated knee. The clinician informs you that his foot is perfused but has sluggish capillary refill. He has a palpable dorsalis pedis pulse but no posterior tibial pulse. Which of the following would you expect to find on the angiogram?
    A. Severed anterior tibial artery
    B. Severed tibioperoneal trunk
    C. Traumatic popliteal artery dissection
    1). Atheromatous plaque at the peroneal origin E. Normal anatomy
A
  1. B. Severed tibioperoneal trunk
    The most common anatomical configuration of the popliteal trifurcation is the anterior tibial (AT) artery branching off first and terminating as the dorsalis pedis artery. The tibioperoneal trunk arises distal to the AT origin and gives rise to the peroneal artery and posterior tibial (PT) arteries.
54
Q
  1. A 13-year old post-pubescent girl presents to the emergency department with acute abdominal pain sited predominantly within the right iliac fossa. An ultrasound scan is performed. This reveals an echogenic mass within the right side of pelvis measuring approximately 4 cm. The sonographer thinks it is adjacent to and inseparable from the right ovary. What is the most likely diagnosis?
    A. Acute appendicitis
    B. Ovarian dermoid
    C. Ovarian torsion
    D. Ectopic pregnancy
    E. Haemorrhagic ovarian cyst
A
  1. E. Haemorrhagic ovarian cyst
    Acute pelvic pain in adolescent girls is a common problem, but ultrasound scanning is very useful in differentiating the many possible causes. Haemorrhagic ovarian cysts are a common cause of pelvic pain in adolescent girls and appear as an echogenic mass in relation to the ovary.
    Acute appendicitis is likely to occur as a blind-ending tubular structure. This may appear like a ‘target lesion’ in cross section and there may be fluid/collection adjacent to it. An acute appendix should be clearly distinct from the right ovary.
    Ovarian dermoids arc usually predominantly fat filled and therefore echogenic on ultrasound, but these tend to be a painless, incidental finding.
    Ovarian torsion can certainly produce an echogenic mass within the right pelvis, but litis is less common than haemorrhagic cysts and would not appear distinct from the ovary.
    Ectopic pregnancy usually appears as a ‘doughnut’-shaped complex mass in relation to one of the uterine tubes; a foetal heartbeat may be present.
55
Q
  1. A young adult presented with complex partial seizures and amnesia. His MRI scan demonstrated T2 hyperintensity within the medial right temporal
    lobe, loss of hippocampal head digitations and dilatation of the ipsilateral temporal horn of lateral ventricle. Which of the following is the most likely diagnosis?
    A. Herpes simplex encephalitis
    B. Choroidal fissure cyst
    C. Mesial temporal sclerosis
    D. Early-onset Alzheimer’s disease
    E. Post-seizure appearance
A
  1. C. Mesial temporal sclerosis
    Mesial temporal sclerosis typically demonstrates an atrophic, T2 hyperintense hippocampus on imaging. Other pathology afFecting the temporal lobe including infection and tumour would cause enlargement of the hippocampus. Alzheimer’s dementia results in medial temporal atrophy, but T2-weighted hyperintensity is not a feature. Herpes encephalitis is caused by Herpes simplex virus (HSV) type I in adults and HSV type II in children. It involves the limbic lobe (hypothalamus, parahippocampal gyrus, cingulate gyrus); and typically appears bilateral but asymmetric. Gyriform enhancement and haemorrhage are late features. Choroidal fissure cyst is a benign incidental finding, similar to arachnoid cysts in the brain. These occur in the region of the hippocampal fissure.
56
Q
  1. A 32-year-old man presented to the A&F, department after injuring his great toe. No fracture was demonstrated on plain X-ray performed, but incidental note was made of a flattened and sclerotic metatarsal head with subchondral collapse compatible with chronic changes of avascular necrosis (AVN). Which metatarsal is the most common site of Freiberg disease?
    A. First
    B. Second
    C. Third
    D. Fourth
    E. Fifth
A
  1. B. Second
    Frieberg disease is essentially osteochondrosis of the metatarsal head, of which the 2nd metatarsal is most commonly affected, and less commonly in the 3rd and 4th metatarsal. Ten percent of cases are bilateral. MRI findings of subchondral fracture of the metatarsal head with severe marrow oedema-like pattern suggest early-stage changes. Metatarsal head collapse with subchondral sclerosis and mild or absent marrow oedema-like pattern suggests late-stage changes.
57
Q
  1. A 39-year-old woman taking human menopausal gonadotrophins presents with pelvic pain, bloating and weight gain. On transvaginal ultrasound, both ovaries are enlarged and contain multiple bilateral cysts, some of which are 8 cm. There is also free fluid within the pouch of Douglas and surrounding the uterus. Which of the following is the most likely diagnosis?
    A. Large corpus luteal cysts
    B. Endometriomas
    C. Polycystic ovarian syndrome
    D. Ovarian hyperstimulation syndrome
    F. Ovarian torsion
A
  1. D. Ovarian hyperstimulation syndrome
    Human menopausal gonadotrophins are an infertility drug, which contains follicle-stimulating hormone and luteinizing hormone, derived from the urine of postmenopausal women and given as an injection. One of its main side effects is ovarian hyperstimulation syndrome, which can also occur because of high levels of beta-human chorionic gonadotrophin seen in multiple pregnancies and hydatidiform moles. Typical symptoms are those mentioned in the question, vomiting and nausea. Corpus luteal cysts are singular and usually unilateral. Ovarian torsion is more common in infertility treatment but is usually unilateral, with a very oedematous ovary containing small, subcentimetre cysts arranged around its periphery. Polycystic ovarian syndrome causes bilaterally increased ovarian volume, but cysts are usually small and subcentimetre. Endometriomas usually contain low-level echoes.
58
Q
  1. You arc asked to perform an ultrasound scan on an asymptomatic 30-year-old pregnant woman 4 weeks after the obstetrician incidentally notes a 3 cm hyperechoic lesion in the right lobe of the liver. On today’s scan, you notice a 7 cm hyperechoic lesion involving segments VI & VII of the liver and subcapsular fluid. What is the most likely diagnosis for this lesion?
    A. Focal nodular hyperplasia
    B. Liver haemangioma
    C. Metastasis
    D. Hepatic adenoma
    E. Hepatic abscess
A
  1. D. Hepatic adenoma
    Hepatic adenomas are benign encapsulated liver lesions, which are usually 8-15 cm in diameter but can grow up to 30 cm. They are associated with the oral contraceptive pill and can often increase in size during pregnancy. They appear as hyperechoic lesions on ultrasound and are usually of low density on plain computed tomography and enhance in the arterial phase with rapid washout. They have a risk of haemorrhage and, if rupture occurs, can develop into a subcapsular haematoma. Focal nodular hyperplasia, haemangiomas and metastases would not be expected to grow this fast. Hepatic abscesses can grow quickly but the patient would be unwell, especially with spread into a subcapsular collection.
59
Q
  1. A 72-year-old man presents with recurrent episodes of flash pulmonary oedema. Magnetic resonance angiography shows bilateral renal artery stenosis with atherosclerotic disease at both renal artery ostia. Which of the following do you advise?
    A. No endovascular option
    B. Bilateral renal artery radiofrequency (RF) denervation
    C. Bilateral renal artery angioplasty and stenting
    D. Continue with best medical management
    E. Refer to vascular surgeons for bilateral renal artery endarterectomy
A
  1. C. Bilateral renal artery angioplasty and stenting
    The results of the ASTRAL trial have shown no benefit of endovascular treatment when compared with best medical management of hypertension. However, recurrent flash pulmonary oedema is a good indication for intervention. Surgical endarterectomy is a procedure that carries huge risks and has no role in this case. Radiofrequency (RF) denervation is a relatively new method. The renal arteries are catheterised via a common femoral artery approach. Using an RF generator, the artery is denervated, excluding the kidneys, from the sympathetic control.
    The long-term results are unknown.
60
Q
  1. A 7-year-old boy who is otherwise well is being investigated for chronic right sided hip pain. Which of the following diagnoses is most likely?
    A. Septic arthritis
    B. Perthes disease
    C. Slipped upper femoral epiphysis
    D. Juvenile rheumatoid arthritis
    E. Developmental dysplasia of the hip
A
  1. B. Perthes disease
    There are a large number of potential causes for hip pain in children. However, many of the conditions have common presenting age ranges, which makes creation of a differential diagnosis difficult.
    Perthes disease is an idiopathic condition where avascular necrosis affects the femoral head leading to chronic destruction of one or both heads of femur. It tends to affect boys between the ages of 5 and 8 years.
    Septic arthritis occurs at any age but would not be expected to present as a chronic finding, especially as many children have associated septicaemia.
    Slipped upper femoral epiphysis affects adolescent children (12-15 years) and is usually associated with childhood obesity or in those who perform high impact activity.
    Juvenile rheumatoid arthritis is similar to the adult form of the disease in that it is idiopathic but tends to affect the large joints of which the hip is not the most common. In addition, there are often other systemic features of the disease as well as multiple joint involvement. Finally, this condition tends to present in younger children (1-3 years).
    Developmental dysplasia of the hip presents in infancy but if undetected will lead to chronic hip problems.
61
Q
  1. Which one of the following is the incorrect association?
    A. Astrocytoma - Intramedullary
    B. Abscess - Extradural
    C. Nerve sheath tumour - Intradural extramedullary
    D. Dermoid - Extradural
    E. Meningioma - Intramedullary
A
  1. E. Meningioma Intramedullary
    Lesions of the spinal canal can be divided into three categories:
    Intramedullary lesions:
    Ependymoma, astrocytoma, ganglioglioma, haemangioblastoma, PNET, metastasis, lymphoma, lipoma, epidermoid, abscess, vascular malformation, MS plaques, infarction, myelomalacia, sarcoidosis
    Intradural extramedullary.
    Nerve sheath tumour, meningioma, lipoma, dermoid, metastasis from outside CNS, ependymoma of filum terminale, arachnoid cyst, neuroenteric cyst
    Extradural:
    Dermoid, epidermoid, lipoma, lymphoma, metastasis, drop metastasis, meningioma, disc material, arachnoid cyst, infection
62
Q
  1. A 10 year old girl of Jewish descent presents with pain in her left thigh. A radiograph revealed diffuse medullary osteoporosis, a ‘flask-shaped’ distal femur, a serpentine area of sclerosis within the femoral metaphysis, and a sharply circumscribed endosteal lytic lesion in the distal femur with a pathological fracture. Clinical examination reveals splenomegaly. Which of the following is the most likely diagnosis?
    A. Thalassemia
    B. Osteopetrosis
    C. Diaphyseal aclasis
    D. Gaucher disease
    E. Rickets
A
  1. D. Gaucher disease
    Although the Ashkenazi Jews are particularly predisposed to this hereditary condition, Gaucher disease is not confined to any particular ethnic group or sex. Splenic enlargement is detected in up to 95% of cases. There is abnormal modelling of the distal femur and proximal tibia secondary to marrow infiltration, leading to an ‘Erlenmeyer flask’ deformity. This feature is, however, not diagnostic for Gaucher disease and may be seen in all the other answer options. Diffuse medullary osteoporosis, bone infarcts and sharply circumscribed endosteal lytic lesions (owing to marrow replacement) can also be seen. The combination of Jewish ancestry, the radiographic features described and splenomegaly, makes Gaucher disease the most likely diagnosis.
63
Q
  1. A 29-year old woman attends for a transvaginal ultrasound for dyspareunia. While passing the probe, you notice an anechoic cystic lesion in the lower vagina within the posterolateral wall. Which of the following is the most likely diagnosis?
    A. Lipoma
    B. Nabothian cyst
    C. Gartner duct cyst
    D. Bartholin’s cyst
    E. Squamous cell carcinoma
A
  1. D. Bartholin’s cyst
    Bartholin’s glands secrete mucous to lubricate the vagina and are equivalent to the male Cowper glands (adjacent to urethra, producing pre-ejaculate). They lie within the posterolateral portion of the lower vagina. Bartholin’s cysts occur when the small duct becomes obstructed; superadded infection can occur, leading to an abscess. Nabothian cysts are small mucous-filled cysts on the cervix and are very common. Gartner duct cysts are a Mullerian duct remnant and are uncommon; they have a typical location within the anterolateral proximal third of the vagina. Squamous cell carcinoma would usually appear as a thickened vaginal wall on ultrasound, if seen at all. Usually, the proximity of the probe to the lesion can make it difficult to assess, so this is better demonstrated with magnetic resonance imaging. A lipoma would have the appearance of other superficial lipomas and characteristically would be hyperechoic
64
Q
  1. A previously healthy 36-year-old woman presents with acute hepatic failure of unknown cause. Her international normalized ratio (INR) is 2.6 and fails to normalise despite treatment. An ultrasound scan shows ascites. The liver team asks you to consider a liver biopsy. Which of the following is the most appropriate reply?
    A. Unable to perform because of INR and ascites
    B. Advise the patient of the increased risk of bleeding and go ahead with liver biopsy
    C. Drain the ascites and the perform the biopsy
    D. Perform transjugular liver biopsy
    E. Perform transarterial liver biopsy
A
  1. D. Perform transjugular liver biopsy
    This is a commonly occurring scenario. Liver biopsy is contraindicated in the presence of ascites and coagulopathy. The transjugular approach is a good alternative where available. The right internal jugular vein is punctured and a catheter is advanced into a hepatic vein, usually the right. Through this, a long specially designed biopsy needle is introduced and a biopsy is taken. The method is used for generalised liver disease and not for a focal lesion. An alternative method is the plugged liver biopsy; here the biopsy is done over a sheath, and coils are dropped after the core is taken to plug the hole.
65
Q
  1. A 79-year old diabetic patient presents with right short distance calf claudication. Magnetic resonance angiography shows focal stenosis of the right popliteal segment, and a decision is made to proceed to angiography and endovascular treatment Which of the following is the best approach for catheter angiography?
    A. Right common femoral artery (CFA), retrograde
    B. Left CFA, retrograde C Right CFA, antegrade
    D. Right popliteal puncture under ultrasound scan
    E. Left CFA, retrograde
A
  1. C. Right CFA, antegrade
    The ‘down the leg’, antegrade, approach via a right common femoral artery puncture is the easiest and most direct approach. Retrograde puncture is usually used for angiography and intervention at the iliac systems, aorta and aortic branches. Ultrasound scan is commonly used to visualise the vessels during puncture. The puncture can also be safely performed using anatomical landmarks and feeling for the femoral pulse. The CFA can be found in the mid point between the anterior superior iliac spine and the symphysis pubis. By using USS, the local anaesthetic can be accurately placed around the vessel, ensuring good analgesia. The profunda femoris artery can also be visualised and hence minimise the chance of accidental puncture.
    The popliteal artery can be punctured using ultrasound guidance at the popliteal fossa, but it would not be preferred in this case.
66
Q
  1. A 2 year old boy is referred for a painless lump in the right supraorbital region. The lesion is fluctuant on clinical examination. On ultrasound, it demonstrates low internal echoes and scalloping of underlying bone. What is the most likely diagnosis?
    A. Angular dermoid
    B. Haemangioma
    C. Neurofibroma
    D. Lymphangioma
    E. Aneurysmal bone cyst
A
  1. A. Angular dermoid
    The location of this lesion at the angle of the eye brow with ultrasound imaging features such as contiguous bone scalloping, fluctuance and presence of mobile debris within the lesion is reassuring and points to a diagnosis of an angular dermoid. T1 hyperintensity demonstrated on MRI consistent with a fat signal is diagnostic of this lesion. As the lesion may also contain debris, this may result in an inhomogeneous signal on MRI. A haemangioma would be expected to demonstrate flow voids and increased vascularity on ultrasound and MRI. The location is very unusual for a lymphangioma.
67
Q
  1. Which of the following is an absolute contraindication to vertebroplasty?
    A. Symptomatic vertebral haemangioma
    B. Metastasis
    C. Multiple myeloma
    D. Infection
    E. Severely painful osteoporosis
A
  1. D. Infection
    Percutaneous cementoplasty (vertebroplasty) with acrylic cement, PMMA (polymethylmethacrylate), is a procedure aimed at preventing vertebral body collapse and pain in patients with pathologic vertebral bodies.
    Indications:
    * Symptomatic vertebral angioma.
    * Painful vertebral body tumours and acetabular tumours.
    * Severe painful osteoporosis with loss of height and/or with compression fractures of vertebral bodies.
    Contraindications:
    * Haemorrhagic diathesis.
    * Infection.
    * lesions with epidural extension. These require careful injection to prevent epidural overflow and spinal cord compression by the cement or displaced epidural tissue.
    The absolute contraindications are haemorrhagic diathesis and infection. Patients with more than five metastases or diffuse metastases are not candidates for vertebroplasty.
68
Q
  1. An elderly man presents to his GP with pain in his left middle finger in the region of the nail bed. This has worsened over the past 2 months. Plain film reveals a destructive lytic lesion within the distal phalanx of his left middle finger. What is the likely diagnosis?
    A. Breast cancer metastasis
    B. Renal cancer metastasis
    C. Lung cancer metastasis
    D. Gastric cancer metastasis
    E. Pancreatic cancer metastasis
A
  1. C. Lung cancer metastasis
    The most common sites for bone metastases arc bones that contain red bone marrow.
    Hence, hone metastases have a predilection for the axial skeleton. Peripheral metastases are rare, and 50% of these are caused by lung cancer. The other options are also known causes of lytic metastases, but would be less likely to affect the hands.
69
Q
  1. A 34 year-old pregnant woman with severe abdominal pain attends for an MRI scan after an ultrasound showing a uterine mass. As well as the normal gestational sac, on T1W imaging there is a low-signal mass noted within the posterior myometrial wall, which displaces the uterine cavity anteriorly. This mass has a high T1 signal intensity rim. On T2W images, this lesion is intermediate to low signal and has a low-signal rim. Which of the following is the most likely diagnosis?
    A. Non-degenerated leiomyoma
    B. Hyaline degeneration within a leiomyoma
    C. Red degeneration within a leiomyoma
    D. Myxoid degeneration within a leiomyoma
    E. Cystic degeneration within a leiomyoma
A
  1. C. Red degeneration within a leiomyoma
    Fibroids (leiomyomas) are benign tumours made of smooth muscle. Afro-Caribbean women are more prone than Caucasian women to developing fibroids. Fibroids can outgrow their blood supply and undergo degeneration causing them to look very different on MRI depending on the type of degeneration that they undergo. Typically, a non-degenerated fibroid is a well-defined low-intensity lesion on T1 and T2-weighted images, and enhances post contrast injection. Hyaline degeneration is caused by necrotic central change and increasing collagen content; this causes variable signal on T1-weighted images but low signal on T2-weighted images. With calcific degeneration, the fibroid has similar signal characteristics on T1 and T2 to typical non- degenerative fibroids. Cystic degeneration can produce areas of fluid signal within the fibroid, which does not enhance. Myxoid degeneration results in a very high T2 signal with a little enhancement. Red degeneration is a very rare type of degeneration; it typically causes severe pain and can cause a fever. It is the result of thrombosis of the vessels surrounding the fibroid leading to the high T1 and low T2 signal rim, and it usually occurs in pregnancy.
70
Q
  1. You are performing a tunnelled dual lumen dialysis line via a right jugular approach on
    a 50-year-old with chronic renal failure. As you are peeling the sheath, you hear a hushing sound and the patient suddenly becomes agitated, with an acute decrease in his partial oxygen saturation pressure. Which of the following is the most appropriate immediate management?
    A. Perform needle decompression in the right mid-clavicular line, 2nd interspace
    B. Obtain a chest X-ray
    C. Inject 10 mL of contrast down the sheath and look for extravasation
    D. Put the patient head down and give fast intravenous crystalloid
    E. Administer high-flow oxygen and put the patient in the left lateral position.
A
  1. E. Administer high-flow oxygen and put the patient in the left lateral position.
    This scenario describes an iatrogenic air embolism. This is an uncommon complication that has an incidence of aroimd 0.13%. Treatment includes high-flow oxygen and placing the patient in the left decubitus position. This would allow the air to rise in the right atrium where it can be aspirated using a transjugular approach.
    Needle decompression in the right mid-clavicular line is the treatment for a tension pneumothorax. A chest X-ray might show air in the right heart but will not help in the immediate management. Injecting 10 mL of contrast down the sheath is performed when you are suspecting extravasation and placing the patient head down and giving fast intravenous crystalloid is the treatment for hypotension.
71
Q
  1. The surgeons want you to review the CT of a young man at an X-ray meeting. A young patient had been admitted with acute abdominal pain and intussusception was diagnosed on pre- operative CT scan. Regarding this condition, which one of the following statements is incorrect?
    A. A lead point is rarely the underlying cause in adult cases.
    B. The intussusceptum is the entering limb of bowel.
    C. Ileocolic intussusception is the most common type in children.
    D. The CT appearance is of multiple concentric rings.
    E. Intussusception is the leading cause of acquired bowel obstruction in childhood.
A
  1. A. A lead point is rarely the underlying cause in adult cases.
    Intussusception refers to the telescope-like invagination or prolapse of an intestinal segment mto the lumen of an adjacent loop. Approximately 95% of cases occur in children, usually affecting those under the age of 2 years. A lead point is an unusual finding in this age group, with mucosal oedema and lymphoid hyperplasia secondary to viral gastroenteritis often being the underlying issue. Conversely, while intussusception in adults is uncommon, a lead point is far more commonly present. Lesions such as polyps, small bowel tumours, post-surgical changes and foreign bodies are recognised precipitants. In children, reduction of intussusception using either water or air can be attempted, provided that there is no initial evidence of perforation. A rule of ‘3s’ is well known, whereby a maximum of three attempts at reduction with 3 minutes separating each attempt and a hydrostatic pressure equivalent to an infusion bag 3 feet (91.4 cm) above the patient is observed. Surgical intervention may be required if such techniques are unsuccessful.
72
Q
  1. A 10-year-old girl of African origin presents with colicky right upper quadrant pain and deranged liver function tests. Ultrasound of the abdomen demonstrates acute calculus cholecystitis. What is the most likely diagnosis?
    A. Sickle cell anaemia
    B. Familial hypercholesterolaemia
    C. Inflammatory bowel disease
    D. Choledochal cyst
    E. Thalassaemia minor
A
  1. A. Sickle cell anaemia
    Gallstones are an uncommon occurrence in childhood irrespective of the symptomatology and therefore should always raise the possibility of a background predisposing condition leading to their formation. In the African population, sickle cell anaemia is the most likely diagnosis. One must also consider other conditions such as malabsorption, Crohn disease, thalassemia major (in patients of Mediterranean origin), total parenteral nutrition (TPN), cystic fibrosis and short gut syndrome. The radiologist is often the first to raise the possibility of a predisposing condition. Children are at risk of the same gallstone-related complications as adults, including severe acute pancreatitis, which may be life threatening.
73
Q
  1. A 60-year-old non-smoker presents with a history of increasing headaches and loss of consciousness. An unenhanced CT of the brain is performed which demonstrates high
    attenuation lesions within the cerebellum. All of the following may appear hyperdense on non contrast CT, except
    A. Bronchogenic metastases
    B. Melanoma metastases
    C. Renal cell metastases
    D. Prostate metastases
    E. Thyroid carcinoma metastases
A
  1. D. Prostate metastases
    Brain metastases may be solitary’ or multiple. Some tumours including breast, renal cell, colon and thyroid cancers are more commonly solitary, while others such as lung cancer and melanoma tend to be multiple.
    Imaging characteristics of metastases may suggest an underlying pathologic diagnosis. Metastases that classically haemorrhage include melanoma, choriocarcinoma, renal cell carcinoma and thyroid cancer. Lung metastases are also known to haemorrhage. In the absence of haemorrhage, metastases may be hypodense, isodense or hyperdense compared with the brain. Acutely haemorrhagic metastases appear hyperdense to brain. Melanoma metastases tend to be hyperdense to brain on CT even in the absence of haemorrhage
74
Q
  1. On an AP skull radiograph, there are fractures of the pterygoid plate, maxilla and nasal septum. What is the best classification for this type of injury?
    A. Le Fort I facial fracture
    B. Le Fort II
    C. Le Fort III
    D. Depressed skull fracture
    E. Pyramidal fracture
A
  1. A. Le Fort I facial fracture
    All Le Fort fractures involve the pterygoid plate. In addition, Le Fort I involves maxilla and nasal septum. Le Fort II involves nasal bones, frontal process of maxilla, maxillary sinus, medial and inferior orbital wail. Le Fort III, in addition to II, involves lateral orbital wall and zygomatico-frontal suture. Le Fort II fracture is also called pyramidal fracture.
75
Q
  1. A 56-year-old woman with known pancreatic cancer is brought from home by ambulance with haematemesis and melaena. After stabilisation, she undergoes endoscopy, which reveals isolated gastric fundus varices with a normal oesophagus. She also undergoes abdominal computed tomography, which shows echogenic material in one of the splanchnic veins, consistent with thrombosis. Which of the splanchnic veins is most likely to be associated with isolated gastric fundus varices rather than gastro-oesophageal varices?
    A. Superior mesenteric vein
    B. Inferior mesenteric vein
    C. Portal vein
    D. Splenic vein
    E. Hepatic veins
A
  1. D. Splenic vein
    The splanchnic circulation includes the superior mesenteric vein, inferior mesenteric vein and splenic and portal veins. Hepatic veins do not form part of the splanchnic circulation as they do not drain to the portal vein but drain directly to the inferior vena cava (IVC).
    Chronic thrombosis of portal or superior and inferior mesenteric veins is often associated with gastro-oesophageal varices, owing to collateral circulation to IVC from submucosal veins of the oesophagus. Isolated splenic vein thrombosis causes only gastric fundal varices, formed by short gastric vessels serving as collaterals. This is known as left-sided portal hypertension. Left-sided portal hypertension is often associated with pancreatitis or pancreatic cancer.
76
Q
  1. A 40 year old with recurrent pulmonary emboli is due to have a hip replacement and it is decided to deploy a temporary inferior vena cava (IVC) filter. What is the preferred site of deployment of an IVC filter?
    A. Suprarenal IVC
    B. Infrarenal IVC
    C. Proximal to the clot load, no matter the level
    D. At the confluence of common iliac vessels
    E. At the junction of IVC and right atrium
A
  1. B. Infrarenal IVC
    In the presence of normal anatomy, IVC filters should be placed inferior to the renal veins. This is to avoid potential clot propagation and renal vein thrombosis. If the patient has aberrant anatomy such as double IVC, a single suprarenal filter or twin IVC filters can be placed. Access is via a right internal jugular puncture or a right femoral vein puncture. A cavogram is performed to visualise the renal veins and to look for aberrant anatomy as described above. Indications for IVC filter include deep vein thrombosis where anticoagulation is contraindicated, where the patient is non-compliant with medical treatment or when there is free floating thrombus in the IVC. Retrievable devices are available and should be removed within 14 days from insertion. Pulmonary embolism can still occur despite an IVC filter with an incidence of 2.7%-4%.
77
Q
  1. Which of the following is an appropriate puncture site for a percutaneous nephrostomy in a patient with normal sonographic anatomy? (You anticipate no need for further intervention.)
    A. Posterior calyx, middle/lower pole
    B. Posterior calyx, upper pole
    C. Direct puncture of the renal pelvis
    D. Anterior, middle pole calyx
    E. Anterior calyx, upper pole
A
  1. A. Posterior calyx, middle/lower pole
    When further intervention such as antegrade stent placement or percutaneous nephrolithotomy is anticipated, a posterior upper pole calyx may be appropriate, Effort should be made to puncture below the 12th rib to avoid traversing the pleura. However, this is not always possible and puncture between the 11th and 12th ribs can also be used with care. Direct puncture of the renal pelvis should be avoided as the adjacent vessels here are large. Anterior calyces should also be avoided, as puncturing these from a posterior approach increases the risk of renovascular injury because of the volume of renal parenchyma traversed.
78
Q
  1. An 18-month-old toddler with a history of prematurity presents with an acutely distended abdomen and vomiting. He has not passed flatus for 24 hours. On examination, the referring accident and emergency registrar is concerned that the patient may have a torted testis, which he believes is causing the abdominal pain. An ultrasound demonstrates mixed echogenicity in the right scrotum. The testis could not be identified. What is the most likely diagnosis?
    A. Acute testicular torsion
    B. Testicular carcinoma
    C. Inguinal hernia
    D. Femoral hernia
    E. Sacro-coccygeal teratoma
A
  1. C. Inguinal hernia
    The child in this scenario has acute bowel obstruction, which can sometimes be missed on initial assessment. When assessing plain films, the hernial orifices should be remembered as an important ‘review area’ as inguinal hernia are common and should not be missed, both in children and adults alike. In this case, an ultrasound was performed first but the same principles apply and bowel loops in the scrotum are diagnostic. The radiologist may be the first to raise this possibility.

An incarcerated hernia associated with obstruction or strangulation is an acute surgical emergency and requires urgent surgical intervention. Femoral hernia is uncommon in children. Acute testicular torsion may have a similar presentation and should always be considered. At least some part of a testis should be identifiable in case of torsion, tumour or injury. Non-visualised testis strongly suggests hernia.

79
Q
  1. A 31-year-old presented with a 3-month history of progressive leg weakness, sensory disturbance, urinary hesitancy, urgency and erectile dysfunction. He had treatment for sputum positive pulmonary TB 1 year ago. MRI of the spine shows clumping of nerve roots within the thecal sac and empty thecal sac sign. Intradural cysts are also seen. What is the most likely diagnosis?
    A. Discitis
    B. Cauda equina
    C. Arachnoiditis
    D. Diastomatomyelia
    E. Arachnoid cyst
A
  1. C. Arachnoiditis
    Infectious aetiologies for arachnoiditis include bacterial, viral, fungal and parasitic agents. Non-infectious aetiologies include surgery, intrathecal haemorrhage and the administration of intrathecal agents, such as myelographic contrast media, anaesthetics and steroids. Neoplastic causes include metastasis from breast and lung carcinoma, melanoma and non-Hodgkin lymphoma or direct seeding of the CSF from primary’ CNS tumours like glioblastoma multiforme (GBM), medulloblastoma, ependymoma and choroid plexus carcinoma.
    MRI is the most sensitive modality’ for diagnosis of arachnoiditis. Although arachnoiditis can be present throughout the subarachnoid space, it is most easily seen in the lumbar region where the cauda equina usually floats in ample CSF. In arachnoiditis, the nerve roots are irregularly thickened and clumped together, often stuck to the dura, resulting in an empty thecal sac sign.
80
Q
  1. A healthy 12 year-old boy presents with painful ankles (no history of trauma). A plain radiograph of his ankles demonstrates bilateral symmetrical periosteal reaction of his tibia and fibula. His father reports similar symptoms in adolescence but never had it investigated.
    No treatment is given. The boy had a further plain film performed after a football injury to his ankle at the age of 15 years. No periosteal reaction was seen at this time. Which of the following is the most likely diagnosis?
    A. Thyroid acropathy
    B. Pachydermoperiostosis
    C. Hypertrophic pulmonary osteoarthropathy
    D. Hypervitaminosis A
    E. Venous insufficiency
A
  1. B. Pachydermoperiostosis
    Pachydermoperiostosis is a self-limiting condition with similar radiological findings to those of hypertrophic osteoarthropathy: symmetrical bilateral periosteal reaction. It is seen in adolescents and is of autosomal dominant inheritance pattern. Males tend to be affected more often than females. Differential diagnoses of these radiological findings include thyroid acropachy, hypervitaminosis A, hypertrophic osteoarthropathy, metastases and chronic venous insufficiency. All of the above require treatment before resolution of the periosteal reaction.
    Hypertrophic osteoarthropathy is caused by many pulmonary and non-pulmonary diseases such as benign and malignant thoracic tumours, chronic infection/inflammation (bronchiectasis and lung abscesses), cyanotic congenital heart disease, inflammatory bowel disease and liver cirrhosis. In HO A, resolution of the radiological findings is only really seen following treatment of the underlying condition.
81
Q
  1. A 40-year-old man presents with postprandial abdominal pain and 10 kg weight loss.
    A computed tomography scan with arterial phase imaging shows an indentation at the proximal coeliac axis caused by the low insertion of the median arcuate ligament. Which of the following is the most appropriate treatment option?
    A. Catheter angiography and stenting
    B. Open surgical management
    C. Refer for laparoscopic division of the ligament
    D. Catheter angiography and stenting
    E. No indication for treatment
A
  1. C. Refer for laparoscopic division of the ligament
    The symptoms and radiological findings are consistent with median arcuate ligament syndrome, the compression of the coeliac axis by the median arcuate ligament of the diaphragm. The median arcuate ligament passes superior to the coeliac axis in 10%-24% of the population. It can cause haemodynamically significant compression in a small subgroup. Treatment is controversial but benefit can be seen in cases where there is weight loss, post-stenotic dilatation along with postprandial pain. Laparoscopic ligation is the preferred method.
82
Q
  1. Which one of the following is not an indication for upper urinary tract drainage (nephrostomy or stent) in the presence of renal failure and hydronephrosis?
    A. Retroperitoneal fibrosis
    B. Mid-ureteric calculus
    C. High-pressure chronic retention
    D. Mid-ureteric stricture
    E. Advanced pelvic malignancy
A
  1. C. High-pressure chronic retention
    High-pressure chronic retention is a cause of bilateral hydronephrosis. It is usually caused by prostatic hypertrophy. The patient typically has a non-compliant bladder with incomplete emptying, although its overall capacity is often normal or may even be below normal. The high pressure in the bladder causes functional obstruction of the upper tracts, and hence causes upper
    tract dilatation and impairment of renal function. Treatment is by catheterisation, thus decompressing the bladder and upper tracts, and interval transurethral resection of the prostate if there is any recoverable bladder function. The other answers are valid indications for upper tract de-obstruction.
83
Q
  1. A 65-year-old female patient with known disseminated renal cell cancer presents with increasing abdominal pain and distension. A computed tomography (CT) scan is performed which reveals moderate ascites and mild hepatomegaly. Portal venous phase images show geographic liver and hyperenhancement of a normal-size caudate lobe. On delayed images, a characteristic ‘flip-flop’ pattern of enhancement is observed. Which of the following is the most likely underlying diagnosis?
    A. Portal vein thrombosis
    B. Acute Budd-Chiari syndrome
    C. Liver congestion secondary to fluid overload
    D. Peliosis hepatis secondary to chemotherapy
    E. Chronic Budd-Chiari syndrome
A
  1. B. Acute Budd-Chiari syndrome
    Acute hepatic venous obstruction (Budd-Chiari syndrome) can be associated with disseminated malignancies. In addition to ascites and hepatomegaly, characteristic CT features are geographic liver (mottled liver enhancement) and a flip-flop pattern of enhancement (early-phase hyperenhancement of the central area of the liver with peripheral hypoenhancement, which reverses on delayed imaging with central hypoenhancement and peripheral hyperenhancement). The caudate lobe is spared as it has direct venous drainage to the inferior vena cava, bypassing hepatic veins. In the acute phase, the liver remains normal in size, but with chronic disease there is hypertrophy of the caudate lobe, which takes over the majority of liver function, thereby helping differentiation between acute and chronic Budd-Chiari syndrome.
    Peliosis hepatis may also be associated with disseminated malignancy, but CT demonstrates multiple spherical lesions with centripetal or centrifugal enhancement because of the presence of blood filled cavities.
84
Q
  1. A 47 year-old obese male patient with long-standing fatty liver infiltration and a strong family history of hepatic cirrhosis underwent liver biopsy, which confirmed the diagnosis of non alcoholic steatohepatitis (NASH). He is to be followed up with imaging to exclude progression to cirrhosis. Which radiological feature is not typical for NASH?
    A. Low attenuation of the liver on computed tomography
    B. High signal of liver on T1W magnetic resonance imaging
    C. Irregular liver contour
    D. Hepatomegaly
    E. High echogenicity of the liver parenchyma on ultrasound
A
  1. C. Irregular liver contour
    Non-alcoholic liver steatohepatitis is the second stage of non-alcoholic fatty liver disease (NAFLD), a very common liver disease that affects about 30% of the western population. The most common cause for this disease is obesity and hypertriglyceridemia. The first stage of NAFLD - hepatosteatosis is potentially reversible but when untreated, 60% progress to NASH, which also has features of fibrosis and inflammation. Hepatosteatosis and NASH are indistinguishable radiologically and NASH can only be diagnosed on biopsy, which is performed in high risk patients. Progression to cirrhosis is observed in 10% of patients with NASH, where a small shrunken liver with irregular contour and secondary features of cirrhosis are observed.
85
Q
  1. A 70-year- old woman has a computed tomography scan for diverticulitis. An incidental
    18 mm aneurysm in the mid-portion of the splenic artery is identified on the scan. Which of the following is the most appropriate treatment for this patient?
    A. Catheter angiography
    B. Catheter angiography and coil embolisation
    C. Catheter angiography and stenting
    D. Percutaneous thrombin injection
    E. Conservative management and repeat scan in 6 months
A
  1. E. Conservative management and repeat scan in 6 months
    Splenic aneurysms have an incidence of around 0.1% at autopsy. Indications for treatment include portal hypertension, size >25 mm, female patient of child-bearing age and pseudo- aneurysm. There is an increased risk of rupture in pregnancy. Interventional treatment is usually by coil embolisation although stent grafting and even transabdominal thrombin injection have been described.
86
Q
  1. Which of the following is not a feature of holoprosencephaly?
    A. Single ventricle
    B. Fused thalami
    C. Absent corpus callosum
    D. Tectal beaking
    E. Hypoplasia of the optic nerves
A
  1. D. Tectal beaking
    Holoprosencephaly results from a lack of normal cleavage of the forebrain. The septum pellucidum is always absent in this condition. Holoprosencephaly may be divided into alobar, semilobar and lobar forms depending on the degree of abnormality. Tectal beaking is a feature of Chiari II. Single ventricle, fused thalami, absent corpus callosum and hypoplasia of the optic nerves are all features of the various forms of holoprosencephaly. In its mildest form, lobar holoprosencephaly, absence of the septum pellucidum may be the only abnormality. The lobar form may be associated with septo-optic dysplasia and the two conditions overlap. The degree of facial abnormality and mental retardation mirrors the severity of the intracranial abnormality
87
Q
  1. A 69-year-old man with a chronic headache underwent an MRI brain and MR cerebral venogram, which showed a chronic thrombosis of the left transverse and sigmoid sinuses with evidence of recanalisation. As far as diagnosis of cerebral venous sinus thrombosis is concerned, which of the following statement is false?
    A. Polycythemia can cause positive delta sign.
    B. Unenhanced MRI is more sensitive than unenhanced CT.
    C. Slow flow mimics sinus thrombosis on contrast-enhanced MRV.
    D. Turbulent flow is more of a pitfall on TOF MRV.
    E. Irregular sinus and intrasinus channels suggest recanalisation
A
  1. C. Slow flow mimics sinus thrombosis on contrast-enhanced MRV.
    The classic finding on unenhanced CT is a hyperattenuating thrombus in the occluded sinus (delta sign). However, this is seen in only 25% cases. Increased attenuation in the venous sinuses may also be seen in patients with dehydration, an elevated haematocrit level, or a subjacent subarachnoid or subdural haemorrhage. On contrast-enhanced CT/MRI, the empty delta sign is seen, a central intraluminal filling defect that represents a thrombus surrounded by contrast enhanced dural collateral venous channels and cavernous spaces within the dural envelope. Unenhanced MRI is more sensitive for the detection of venous thrombi than is unenhanced CT. The absence of a flow void and the presence of altered signal in the sinus is a primary finding. Slow or turbulent flow also may cause a signal intensity alteration in the sinus.
    The signal intensity of venous thrombi on Tl- and T2-weighted MR images varies according to the interval between thrombus formation and the time of imaging. Time-of-flight (TOF) MR venography is the method most commonly used for diagnosis. Contrast-enhanced MRV utilises the paramagnetic effect of intravenous gadolinium to shorten Tl relaxation time and provide positive intravascular contrast enhancement. Small-vessel visualisation is improved at contrast- enhanced MRV. Depiction of the dural sinuses is also superior with contrast enhanced MR venography because of a decrease in the effects of turbulent/slow flow on vessel contrast.
    An irregular appearance of the sinus with multiple intrasinus channels and dural collateral vessels on MRV is characteristic of incomplete recanalisation. Complete recanalisation occurs more often in superior sagittal and straight sinus thrombosis than transverse and sigmoid sinuses after anticoagulation therapy.
88
Q
  1. Which of the following statements is incorrect as regards the anatomy and imaging findings of the seminal vesicles and vasa deferens?
    A. The seminal vesicles are located posterior to the bladder and distal ureters.
    B. Normal seminal vesicles appear hypointense and hyperintense on T1W imaging and T2W imaging respectively.
    C. The seminal vesicle joins the distal portion of the vas deferens, becoming the ejaculatory duct, and then drains into the prostatic urethra.
    D. The distal portions of the seminal vesicles and vas deferens are intraperitoneal.
    E. The seminal vesicles initially increase in volume with age, but then steadily reduce in size with advancing age.
A
  1. D. The distal portions of the seminal vesicles and vas deferens are intraperitoneal.
    The seminal vesicles arc paired, elongated septated structures that lie above the prostate and abut the posterior wall of the bladder. Their fluid accounts for around 50% 80% of ejaculate. The seminal vesicle joins the distal vas deferens, forming the ejaculatory duct, which drains into the prostatic urethra at the verumontanum. The proximal portion of the vas deferens lies within the spermatic cord. The seminal vesicles and vasa deferentia are extraperitoneal structures. Seminal vesicle abnormalities are invariably found as incidental findings on imaging studies, although patients may present with haematospermia or infertility. Primary tumours of the seminal vesicle arc very rare indeed, with the majority of neoplasia occurring from local spread of prostate, bladder or rectal malignancies.
89
Q
  1. A 65-year-old smoker presents with an achy pain around both his ankles. The GP orders ankle radiographs to look for degenerative change. The report comes back describing ‘smooth, lamellar periosteal reaction with new bone formation in the distal diametaphyses of both tibiae’. Which of the following is the most likely cause?
    A. Systemic lupus erythematosus
    B. Low-grade chronic osteomyelitis
    C. Rheumatoid arthritis
    D. Hypertrophic osteoarthropathy
    E. Reiter syndrome
A
  1. D. Hypertrophic osteoarthropathy
    Hypertrophic osteoarthropathy, or HOA, is characterised by smooth periosteal new bone formation, usually in the diametaphysis of the long bones. The tibia, fibula, radius and ulna are most commonly involved in -80% of cases with patients usually describing painful, swollen joints. When present, caution must be given to look for an underlying thoracic cause of which the most common aetiology is bronchogenic carcinoma. In such instances, it is commonly referred to as hypertrophic pulmonary osteoarthropathy, or HPOA. Extrathoracic causes of HOA include inflammatory bowel disease and liver cirrhosis.
90
Q
  1. You are performing a varicocele embolisation. The patient suddenly develops left loin pain. He remains haemodynamically stable, but you notice contrast extravasation proximal to the coils. Which of the following is the most appropriate management?
    A. Inject polyvinyl alcohol (PVA) at the extravasation site
    B. Stent grafting to exclude the extravasation site
    C. Alert the surgical team - need to proceed to laparotomy
    D. Take the patient for a computed tomography scan of the abdomen/pelvis
    E. Conservative management
A
  1. E. Conservative management
    Varicocele is a commonly identified correctable cause of male factor infertility. Surgical correction has a failure rate of less than 5%. An alternative to surgery is the selective catheterisation and embolisation of the gonadal vein. The gonadal vein is catheterised via a common femoral vein puncture and embolic material is introduced. Indications include symptomatic varicocele, recurrence of varicocele post treatment and varicocele with associated infertility. Complications include pain, recurrence and reaction to iodinated contrast. Rupture of the testicular vein is a known complication, but it needs no specific treatment. Unlike surgery, embolisation is not associated with postoperative hydrocele or testicular loss from inadvertent injury to testicular artery. However there are case reports of renal loss from coil migration.
91
Q
  1. Chest radiograph of a 12-year-old boy shows a cystic lesion with air-fluid level in the right upper lobe. CT scan confirms the presence of a thin walled cystic lesion. Rest
    of the lungs are clear. There is no lymphadenopathy in the chest. Quantiferon test was negative, and there are no features of infection or signs of inflammation. What is the diagnosis?
    A. TB
    B. Intrapulmonary bronchogenic cyst
    C. Hydatid cyst
    D. Infected bulla
    E. Congenital lobar emphysema
A
    1. Intrapulmonary bronchogenic cyst
      Bronchogenic cysts (BCs) are congenital lesions. They are usually found in the mediastinum or pulmonary parenchyma and, less commonly, cysts may be found in the neck, pericardium, pleura, diaphragm or abdominal cavity. Intrapulmonary cysts are most common in the lower lobes. Intrapulmonary BCs are usually sharply defined, solitary, non-calcified, round or oval opacities confined to a single lobe. These can present as a homogeneous water density, an air-filled cyst, or with an air-fluid level. Signal on MRI depends on the content, and fluid-containing lesions are low on T1-weighted and high on T2 weighted images; however, proteinaceous content makes them high on T1-weighted imaging.
      The differential diagnosis of intraparenchymal BCs must include acquired cystic lesions, such as a lung abscess, a hydatid cyst, infection with nocardia, an infected bulla, congenital lobar emphysema, fungal diseases and tuberculosis, especially when the lesions manifest as air filled or have an air-fluid level.
92
Q
  1. A 40-year-old man presents with acute onset of III cranial nerve palsy. The unenhanced CT shows subarachnoid blood. Where is the aneurysm likely to be?
    A. Anterior communicating cerebral artery
    B. Anterior cerebral artery
    C. Middle cerebral artery
    D. Posterior cerebral artery
    E. Posterior communicating cerebral artery
A
  1. E. Posterior communicating cerebral artery
    The third nerves exit the brain stem medial to the cerebral peduncles, and course forward and laterally in the interpeduncular cistern between the posterior cerebral arteries-posterior communicating arteries (PcomA) above and superior cerebellar arteries below. The pupillary fibres are located dorsomedially and peripherally at this segment.
    Common pathologies involving this segment include aneurysm at PcomA or basilar tip, dolichoectatic vessels, microvascular ischaemia, SAH, meningitis, neoplasms (leukaemia, lymphoma, neurogenic tumours and leptomeningeal carcinomatosis), inflammatory disease (neurosarcoidosis, Wegener’s granulomatosis), demyelinating disease, transtentorial herniation (from supratentorial tumour, haemorrhage or brain swelling) and head trauma with nerve avulsion (usually at the posterior petroclinoid ligament where the third nerve is stretched because of downward displacement of the brain stem at the time of impact).
93
Q
  1. A 7-year-old girl presents with abdominal pain, which has come on and off over the past year, and failure to thrive. An X-ray performed revealed dense metaphyseal bands in both lower limbs. The blood profile was unremarkable. What is the most likely diagnosis?
    A. Leukaemia
    B. Lead poisoning
    C. Congenital rubella
    D. Osteopetrosis
    E. Osteopathia striata
A
  1. B. Lead poisoning
    The differential list for dense metaphyseal bands is plenty but includes normal variant in neonates, growth acceleration lines (usually in patients with chronic illnesses like diabetes, asthma), treated rickets, chronic anaemia, scurvy and lead poisoning. Recurrent abdominal pain is associated with lead poisoning. None of the other conditions, except for leukaemia, is associated with abdominal pain, but the dense metaphyseal band sign is acknowledged only in treated leukaemia rather than active leukaemia. Although the blood profile is often normal, the patient may sometimes have a mild microcystic, microchromic anaemia
94
Q
  1. A 63-year old man is diagnosed with sigmoid adenocarcinoma with no metastatic spread identified on initial staging computed tomography (CT) scan. After a course of neoadjuvant chemotherapy, he is considered for tumour resection and undergoes a repeat CT scan. New hepatic lesions suspicious for metastases are identified. All of the following are criteria for resectability, except:
    A. Less than three hepatic lesions
    B. At least three segments spared from metastatic involvement
    C. No visible nodal involvement
    D. At least one main portal vein branch must be spared
    E. At least one hepatic vein must be spare
A
  1. A. Less than three hepatic lesions
    Liver metastases from colorectal cancer are among the most common hepatic secondary tumours. This is also the only type of metastatic deposit for which resection has shown a survival benefit. If liver metastases from colorectal cancer are to be resected, several radiological criteria must be met.
    There is no strict limit to the number of lesions that can be resected, but it is unlikely that more than six lesions will be removed. Compare this with hepatocellular carcinoma where only a solitary lesion can be surgically explored. If at least three segments of the liver are spared, there are enough hepatocytes to provide liver function following surgery; preoperative selective portal vein embolisation can be used to increase the volume of the remaining liver through hypertrophy.
95
Q
  1. A taxi driver is brought to the accident and emergency department after a high-speed road traffic accident. The fire brigade needed 2 hours to extricate him from his wrecked car. On arrival, he is hypothermic but stable, complaining of severe abdominal pain. An urgent contrast enhanced CT scan is performed and the radiologist on call issues a report describing features suggestive of mesenteric injury with segmental small bowel ischaemia. Which finding is the most specific for a mesenteric injury’ with associated bowel wall ischaemia?
    A. Generalised increased bowel wall enhancement
    B. Patchy and irregular Localised bowel wall enhancement
    C. Localised bowel wall thickening
    D. Decreased or absent bowel wall enhancement
    E. segmental bowel dilatation
A
  1. D. Decreased or absent bowel wall enhancement
    Abnormalities in bowel wall enhancement after administration of IV contrast are very common and important positive findings in the evaluation of abdominal CT in a trauma scenario and should always be assessed. Generalised increased enhancement is typical for bowel wall injury with vascular involvement but without ischaemia, and is thought to be caused by increased permeability of the hypoperfused wall.
    Localised, patchy and irregular hyperenhancement is suggestive but not diagnostic of a fullthickness tear. Areas of decreased or absent enhancement in a segment of a bowel are indicative of bowel ischaemia; lack of enhancement is considered to be highly specific.
96
Q
  1. A 4-week-old neonate is being investigated for neonatal jaundice. Post-feed ultrasound of the abdomen demonstrates a normal looking distended gallbladder and no focal hepatic abnormality. A cholescintigraphy scan, also known as hepatobiliary iminodiacetic add (HIDA) or mebrofenin scan, demonstrates isotope in the urinary bladder at 24 hours. What is the most likely diagnosis?
    A. Choledochal cyst
    B. Transient neonatal hyperbilirubinaemia
    C. Biliary atresia
    D. Idiopathic hepatitis
    E. Cystic fibrosis
A
  1. C. Biliary atresia
    Failure of gallbladder emptying following a feed and failure of isotope (HIDA) excretion into the bowel is a typical finding of biliary atresia. The presence of tracer in the urinary bladder implies that there is no hepatic excretion of tracer, only urinary excretion; this only occurs with sustained systemic levels as HIDA is preferentially excreted hepatically. The gallbladder is not visualised in some cases of high biliary atresia and cystic fibrosis. Transient neonatal hyperbilirubinaemia should resolve spontaneously by 2 weeks of age. Ultrasound imaging may demonstrate a triangular cord or tubular echogenic structure in the porta hepatis owing to fibrous tissue. When identified, this is pathognomonic of biliary atresia.
97
Q
  1. Which of the following is not an angiographic sign of active bleeding?
    A. Contrast extravasation
    B. Vessel spasm
    C. Vessel cut-off
    D. Early venous filling
    E. Vessel dilatation
A
  1. E. Vessel dilatation
    The rest are angiographic signs of active bleeding; vessel dilatation is not.
98
Q
  1. A 5-year-old girl presents with left-sided abdominal pain. On examination, there is a palpable mass in the left flank. Computed tomography demonstrates a well-circumscribed multiseptated cystic renal mass, which is replacing the lower pole of the left kidney. The intervening septa are thick and enhance post-contrast, and the cysts appear to be herniating into the renal pelvis. Which of the following is the most likely diagnosis?
    A. Multicystic dysplastic kidney
    B. Multilocular cystic nephroma
    C. Nephroblastomatosis
    D. Polycystic kidney disease
    E. Mesoblastic nephroma
A
  1. B. Multilocular cystic nephroma
    Multilocular cystic nephroma is a benign renal tumour that occurs in children and, less commonly, adult women. There is no known association with Wilms’ tumour. It is usually a unilateral abnormality that replaces an entire renal pole and presents as a large mass, often around 8-10 cm in diameter. Radiological appearances, while not entirely specific, can help to differentiate this lesion from other renal mass lesions. A sharply well-circumscribed, multiseptated cystic mass is typical, with a thick surrounding capsule. The cysts may appear to herniate into the renal pelvis - an appearance that is relatively specific. Unsurprisingly, these lesions are excised as a definitive radiological differentiation from malignancy is often not possible. Multilocular cystic nephroma can be differentiated from multicystic dysplastic kidney by the presence of normal functioning renal parenchyma and symmetrical renal excretion. Polycystic kidney disease involves the entire kidney, unlike multilocular cystic nephroma, which tends to be localised around a renal pole.
99
Q
  1. A 54-year-old woman is having an MRI for suspected cerebral venous sinus thrombosis. With regard to parenchymal abnormalities associated with venous thrombosis, all are true, except
    A. Parenchymal changes are better depicted on MRI.
    B. DWI helps distinguish between types of cerebral oedema.
    C. Parenchymal changes may not correlate with location of venous occlusion.
    D. Parenchymal abnormalities secondary to venous occlusion are irreversible.
    E. Brain swelling with normal parenchymal signal is well recognised.
A
  1. D. Parenchymal abnormalities secondary to venous occlusion are irreversible. Parenchymal lesions are better depicted and more commonly identified at MR imaging than at
    CT. Focal oedema (without visible haemorrhage) is visible on CT images in approximately 8% of cases and on MR images in 25% of cases. Diffusion-weighted MR imaging techniques allow sub classification of parenchymal abnormalities as either primarily vasogenic oedema (with increased ADC values presumably related to venous congestion) or primarily cytotoxic oedema (with decreased ADC values related to cellular energy’ disruption). Haemorrhage may occur with both types of oedema. In contrast with arterial ischaemic states, many parenchymal abnormalities secondary’ to venous occlusion are reversible. Although parenchymal changes may occur in areas of the brain that are directly drained by the occluded venous sinus, in some patients, the parenchymal changes may not closely correlate with the location of venous occlusion. Parenchymal swelling without abnormalities in attenuation or signal intensity on images may occur in as many as 42% of patients with cerebral venous thrombosis. Sulcal effacement, diminished cistern visibility and a reduction in ventricular size may occur.
100
Q
  1. A 24-year-old man sustained an injury to his wrist following a fall while skiing. There is tenderness in the anatomical snuffbox, particularly on ulnar deviation. Plain radiograph performed in casualty revels a scaphoid fracture. Which one of the following statements regarding fractures of the scaphoid bone is true?
    A. The distal fragment is at risk for avascular necrosis when the waist of the scaphoid is fractured.
    B. They usually are the result of a direct blow to the wrist.
    C. Fractures of the distal pole arc less common than fractures of the proximal pole.
    L). Fractures involving the proximal pole invariably lead to avascular necrosis of the
    proximal fragment.
    E. Fractures through the waist of the scaphoid do not displace because of the strong supporting intercarpal ligaments.
A
  1. C. Fractures of the distal pole are less common than fractures of the proximal pole. Scaphoid fractures arc common injuries but can have long-standing implications if not
    treated in a timely manner because of the risk of avascular necrosis and early development of osteoarthritis in cases of mal- or non-union. Scaphoid fractures most commonly occur within the waist of the scaphoid (70%), with the remaining occurring in the proximal pole (20%) and distal pole (10%). Due to the nature of the scaphoid blood supply, it is the proximal fracture fragment that is at risk of avascular necrosis. A fall on the outstretched hand is the most common mechanism. Fracture displacement and scapholunate ligamentous disruption with widening of scapho-lunate distance, (‘Terry Thomas’ sign) are common associated findings.
101
Q
  1. A 71 -year-old man with biopsy proven Gleason grade 8 adenocarcinoma and a PSA level of 5.65 ng/mL is sent for an MRI prostate for local staging. T2W axial MR image shows that the dominant tumour is within the left peripheral zone extending from the apex to the base with features indicative of extracapsular extension. All the following suggest extracapsular extension, except
    A. Broad contact (>12 mm) of tumour with capsule
    B. Obliteration of rectoprostatic angle
    C. Asymmetry of the neurovascular bundle
    D. Nodes in the perivesical fat
    E. Irregular capsular bulge
A
  1. D. Nodes in the perivescical fat
    The strength of MRI in evaluation of prostate cancer is in the diagnosis of extracapsular and seminal vascular invasion. The criteria used for the detection of extracapsular extension of tumour (on T2-weighted images) include: irregular capsular bulge, obliteration of rectoprostatic angle, asymmetry of the neurovascular bundle, angulation or step-off appearance of the prostate contour,

focal capsular thickening or retraction, broad (>12 mm) capsular tumour contact and breech of capsule with evidence of direct tumour extension.

102
Q
  1. An 8-month old child is investigated for a 5-month history of respiratory symptoms. Chest X-ray shows a right upper zone mass. MRI confirms the presence of a low Tl, high T2 signal, well-defined mass displacing the trachea to the left. There is homogeneous internal signal; the lesion is separate from the spinal canal. There is no associated vertebral anomaly and no widening of the neural foramina. The patient is afebrile. What is the most likely diagnosis?
    A. Neurofibromatosis
    B. Lateral meningocele
    C. Neuroenteric cyst
    D. Lung abscess
    E. Foregut duplication cyst
A
  1. E. Foregut duplication cyst
    Foregut duplication cysts usually occur on the mesenteric aspect of the alimentary canal and may be associated with other gastrointestinal anomalies. The absence of any spinal involvement and the late presentation seen here rule out other differential diagnoses. Other more likely differentials would include bronchogenic cyst and pericardial cyst, and these are often very difficult to differentiate even on computed tomography (CT) and MRI. Duplication cysts occur most commonly in relation to the small bowel, in particular the ileum; the oesophagus is, however, the second most common location. When identified, the ‘muscular rim sign’ on ultrasound is diagnostic. This results from an alternating echogenic inner mucosal lining and a hypoechoic outer rim signal from the mucosal lining.
103
Q
  1. A 10-year-old boy has a 48-hour history of drowsiness and decreasing GCS. In the previous week, he was unwell with a chest infection. The MRI demonstrates multiple areas of hyperintensity on T2 and FLAIR within the white matter. Contrast enhancement demonstrates ‘open ring sign’. What is the most likely diagnosis?
    A. Multiple sclerosis
    B. Acute disseminated encephalomyelitis
    C. Multiple cerebral abscesses
    D. Herpes encephalitis
    E. CMV encephalitis
A
  1. B. Acute disseminated encephalomyelitis
    Acute disseminated encephalomyelitis (ADEM) is a monophasic autoimmune demyelinating disease of the central nervous system that typically follows a febrile infection or vaccination. The disorder is immunologically mediated because of an autoimmune reaction to myelin. Peak incidence is in children aged 3-10 years. Typically, ADEM manifests as multifocal lesions mimicking MS. However, clinical and radiologic criteria distinguish between ADEM and MS.
    ADEM typically meets at least two of the three following criteria: (a) They have clinical symptoms atypical of MS, e.g., altered consciousness, hypersomnia, seizures, cognitive impairment, hemiplegia, quadriparesis, aphasia or bilateral optic neuritis, (b) They have absence of oligoclonal bands in CSF. (c) They have grey matter involvement (basal ganglia or cortical lesions). Imaging features, such as the number of T2 hyperintense lesions and the presence of gadolinium enhancement, oedema and periventricular or brainstem lesions, cannot be used to reliably differentiate between ADEM and MS. Corpus callosal involvement is less frequent in patients with ADEM than in those with MS. Cerebrospinal fluid findings, especially the presence or absence of oligoclonal bands, remain important in the differentiation of patients with MS from those with ADEM. The best criterion is the course of the disease, with a good outcome in 57%-81% of patients with ADEM, with no new lesions and partial or complete recovery of existing lesions on follow up MR images.
104
Q
  1. You notice an area of focal dissection at the site of angioplasty of a left common iliac lesion during your completion run. Which of the following is the next step?
    A. Reinflate the angioplasty balloon
    B. Deploy a stent
    C. Measure the pressure gradient I). Deploy a covered stent
    E. Report the finding and finish the procedure
A
  1. C. Measure the pressure gradient
    There will be a degree of vessel dissection in all vessels after angioplasty. If there is no limitation to flow or significant pressure gradient across the lesion, then the procedure can be regarded as being successful and no further action is necessary. The pressures can be measured by connecting the end of a catheter to a transducer via a three way tab. The catheter is placed through stenosis and the pressures are measured on either side. The difference is the pressure gradient, and it should be less than 10 mmHg.
105
Q
  1. A 32-year-old man who fell of his bike in mid-air during a motor cross championship was brought into casualty with weakness of all four limbs and GCS of 10 out of 15. Initial CT showed
    a fracture dislocation at C7/T1. An MRI was organised given that there were clinical features of acute spinal cord injury. All of the following findings are expected on the MRI, except
    A. Hyperintensity on T2W images
    B. Swelling of the spinal cord
    C. Hypo intensity on T2W images suggesting haemorrhage
    D. Occasional contrast enhancement
    E. Atrophy of the spinal cord
A
  1. E. Atrophy of the spinal cord
    Spinal cord injury can have devastating impacts on patients. Acute causes generally relate to traumatic events and include cord haemorrhage, contusion and transaction. Oedema, swelling and haemorrhage (epi and intradural) are often seen in the context of an acute cord injury.
    Occasionally, there can be increased enhancement in the hyperacute status, usually of the haematoma. Examples of causes of chronic cord myelopathy include chronic spinal stenosis (e.g., from a disc prolapse), from the sequelae of an acute cord injury or from a demyelinating process such as multiple sclerosis. Atrophy of the spinal cord is a common finding in these scenarios.
106
Q
  1. A 21-year-old man, who presented with a growing mass in the left testis of 2-3 months duration with a palpable, non-tender, firm nodule on the surface of the left testicle, on physical examination showed a well circumscribed hypoechoic mass with a concentric lamellar pattern of alternating hyper- and hypoechoic rings on US. What is the diagnosis?
    A. Tunica albuginea cysts
    B. Simple cyst of testis
    C. Epidermoid cyst
    D. Intratesticular varicocele
    E. Intratesticular spermatocele
A
  1. C. Epidermoid cyst
    Epidermoid cysts are also known as keratocysts. They are non tender and usually palpable. The US appearance varies with the maturation, compactness and quantity of keratin present within the epidermoid cyst. A target appearance, a solid mass with an echogenic rim and a characteristic “onion ring” configuration with alternating layers of hyper and hypoechogenicity have been described. These cysts do not show blood flow at Doppler US examination. The constellation of an onion ring configuration, negative tumour marker status and avascularity help differentiate testicular epidermoids from other germ cell tumours.
    The US findings of intratesticular varicocele are similar to those of extratesticular varicocele and include multiple anechoic, serpiginous and tubular structures of varying sizes. Colour flow and duplex Doppler US show a venous flow pattern with a characteristic venous spectral waveform, which increases with the Valsalva manoeuvre. An intratesticular spermatocele is a cystic intraparenchymal lesion that is attached to the mediastinum in the area of the rete testis.
107
Q
  1. A 2 year-old toddler is brought to the accident and emergency department by his mother with constant vomiting. During clinical examination the toddler is restless, dehydrated and shows the signs of peritonitis. The examining doctor also notices multiple bruises of different ages, skin lacerations and scars. Visceral trauma on a background of non-accidental injury (NAI) is suspected. Which of the following is not a common bowel injury associated
    with non-accidental trauma?
    A. Small bowel rupture
    B. Shocked bowel syndrome
    C. Intramural mesenteric haematoma
    D. Boerhaave syndrome
    E. Duodenal contusions
A
  1. D. Boerhaave syndrome
    Non-accidental abdominal trauma is the second leading cause of death from child abuse after head trauma. Presentation is usually delayed with abdominal pain, vomiting, peritonism and signs of obstruction. Non accidental visceral trauma is more common in the age group presented here than is incidental blunt abdominal injury and should always be considered as a differential diagnosis, especially if there are other associated features of NAI. Visceral perforation/ laceration, intramural haematoma and shocked bowel syndrome are all within the spectrum of NAI. Boerhaave syndrome is a complete transmural oesophageal rupture caused by forceful vomiting or impacted food bolus and more commonly affects adults.
108
Q
  1. You are asked to review the chest X-ray of a postoperative patient, now in intensive treatment unit. The patient has a number of lines and tubes. Which of the following positions suggests incorrect placement?
    A. Endotracheal tube with the tip 2 cm above the carina
    B. Central venous catheter with the tip in the junction of the superior vena cava/right atrium
    C. Nasogastric tube with the tip below the diaphragm overlying the stomach
    D. Peripherally inserted tunnelled central venous catheter with the tip in the superior vena cava
    E. Intra-aortic counter pulsation balloon pump in the descending aorta distal to the origin of the left subclavian artery
A
  1. A. Endotracheal tube with the tip 2 cm above the carina
    Close evaluation of any tubes and lines should always be performed to avoid any unnecessary complications and embarrassment. The endotracheal tube should be at least 5 cm above the carina to allow movement of the patient’s body so that it never lies within the right or left main bronchus. If it does, it can cause collapse of the non-intubated lung. Central venous catheters, including peripherally inserted catheters, should have their tip within the superior vena cava or right atrium. Intra-aortic counter pulsation balloon pumps should lie within the descending thoracic aorta with the upper aspect distal to the arch vessels to avoid obstruction of blood flow.
109
Q
  1. A 2-year-boy presents with a 2 week history of melena culminating in an acute episode of bright red blood per rectum. Ultrasound was unremarkable. Upper gastrointestinal endoscopy was negative. Technetium pertechnetate demonstrates increased uptake m the left upper and right lower quadrants. What is the most likely diagnosis?
    A. Acute appendicitis
    B. Intussusception
    C. Meckel’s diverticulum
    D. Gastrinoma
    E. Non-specific inflammatory bowel disease
A
  1. C. Meckel’s diverticulum
    The findings on this technetium pertechnetate scan are typical of a Meckel’s diverticulum containing ectopic gastric mucosa. The uptake in the left upper quadrant should not mislead the radiologist as it is the result of normal tracer uptake in the stomach mucosa. Secretions from ectopic gastric tissue in a Meckel’s diverticulum can cause ulceration of the diverticulum or adjacent small bowel and can lead to bleeding, which, if profuse, can simulate an upper gastrointestinal bleed. Meckel’s diverticulum may simulate acute appendicitis on clinical examination; however, a pertechnetate scan is only performed if a Meckel’s diverticulum is suspected clinically.
110
Q
  1. The following arc appropriate MR spectroscopy findings, except
    A. Canavan disease characteristically demonstrates reduced NAA.
    B. Decreased NAA and higher Cho/Cr ratio signifies high-grade malignancy.
    385
    386
    C- Lipid peaks suggest infracted brain.
    D. Absent choline can distinguish toxoplasmosis from lymphoma.
    E. PML may demonstrate elevated myo-inositol.
A
  1. A. Canavan disease characteristically demonstrates reduced NAA.
    MR spectroscopy (MRS) allows tissue to be interrogated for the presence and concentration of various metabolites.
    MRS can help increase our ability to grade gliomas. As the grade increases, NAA and creatine decrease and choline, lipids and lactate increase. In recurrent tumour, choline will be elevated, whereas in radiation change, NAA, choline and creatine will all be low. With regard to ischaemic and infraction, lactate wall increase as the brain switches to anaerobic metabolism. When infarction takes place, lipids are released and peaks appear. With regard to HIV, choline is low or absent in toxoplasmosis, whereas it is elevated in lymphoma, helping to distinguish the two. PML may demonstrate elevated myoinositol. Canavan disease characteristically demonstrates elevated NAA.
111
Q
  1. A 44-year-old man with chronic back pain underwent a spinal X-ray, which showed extensive marginal osteophytes with flowing ossification along the anterior aspect of more than four vertebrae with no significant facet joint arthropathy. Intervening disc spaces were maintained with no loss of disc height. In the extremities, generally symmetric enthesophytes are seen most commonly in the
    A. Metacarpals, metatarsals and terminal tufts
    B. Calcaneus, patella and olecranon
    C. Lesser trochanter and acetabulum
    D. Medial and lateral epicondyle of humerus
    E. Glenohumeral joint and greater tuberosity
A
  1. B. Calcaneus, patella and olecranon
    The description is characteristic of diffuse idiopathic skeletal hyperostosis, or commonly known as DISH. The predominant finding is the calcification/ossification of the ligament and tendon entheses, most often in the spine. Within the thoracic spine, the ossification tends to occur on the right anterolateral aspect with postulated theories that the pulsations from the adjacent left sided thoracic aorta inhibits the hypertrophic ossification. The most common diagnostic criteria employed is the involvement of at least four contiguous vertebrae (flowing ossification from calcification of the anterior longitudinal ligament) and the absence of sacroiliac inflammatory change and apophyseal joint degeneration. Recent changes have proposed reducing the contiguous vertebral segment involvement to 3 rather than 4. Extraspinal manifestations are again of calcification of the entheses and hypertrophic bone changes, with the most common sites involving the olecranon, patella and calcaneum
112
Q
  1. A 69-year-old man with a history of non muscle invasive urothelial carcinoma of bladder treated with transurethral resection and intravesical BCG therapy presented with a firm palpable nodule on PR examination. MRI showed a low T2 signal lesion with high signal on DW MR, low signal on ADC map and non-enhancement on subtracted contrast-enhanced images in the peripheral gland at 4 ‘o’ clock. The findings were stable on an MR repeated at 9 months.
    A. Prostatic carcinoma
    B. Post-inflammatory scar
    C. Post biopsy haemorrhage
    D. Granulomatous prostatitis
    E. Post-radiotherapy change
A
  1. D. Granulomatous prostatitis
    Granulomatous prostatitis is an inflammatory entity that often presents with a firm nodule on digital rectal examination and elevated prostate-specific antigen, thus clinically mimicking prostate cancer. Possible causes include previous intravesical BCG therapy, TB prostatitis and previous intervention such as TURP, although most cases are idiopathic. On MRI, granulomatous prostatitis may appear as a discrete mass with markedly abnormal T2 signal, DWI and ADC map. Furthermore, there may be associated infiltration of the periprostatic fat by inflammation, thus
    mimicking extraprostatic tumour extension. Currently, histopathologic analysis is regarded as the only means of definitively establishing the diagnosis and thereby excluding the presence of tumour; however, a suggestive history, such as prior BCG therapy, may be useful in prompting consideration of the diagnosis. In addition, the presence on MRI of large areas of non- enhancement, indicative of necrosis within the lesion corresponding with caseous abscess on pathologic evaluation, may suggest the diagnosis. A short-term follow up MRI may he obtained after antimicrobial treatment to assess for therapeutic response.
113
Q
  1. A 14-year-old boy who accidentally fell from a second floor window underwent an urgent computed tomography scan. It revealed diffuse small bowel wall thickening with increased enhancement of bowel wall following intravenous contrast A flattened inferior vena cava and retroperitoneal oedema were also noted in the radiological report. What is the most likely diagnosis?
    A. Small bowel wall contusion
    B. Isolated mesenteric haematoma
    C. Incomplete jejunal wall tear
    D. Complete ileal wall tear
    E. Shocked bowel syndrome
A
  1. E. Shocked bowel syndrome
    Isolated, localised small bowel wall thickening is non-specific but highly suggestive of underlying bowel wall or mesenteric injury in the trauma patient. In all of the above conditions, apart from shocked bowel syndrome, this finding may be present. Diffuse bowel wall thickening is atypical for bowel wall injury. Following trauma, in the presence of increased bowel wall enhancement, a flattened inferior vena cava, hyperenhancement of adrenal glands and pancreatic and/or retroperitoneal oedema, a shocked bowel syndrome (hypoperfusion complex) is the most likely explanation. This must not be omitted in the differential diagnosis as treatment of hypoperfusion complex is conservative and there is no need for diagnostic laparotomy
114
Q
  1. A 65-year-old man with recurrent bouts of acute red hot swollen 1st MTPJ of the right foot showed classic features of gout on plain radiograph. Which one of the following statements regarding the radiographic appearance of gouty tophi is false?
    A. They can contain some degree of calcification.
    B. They are asymmetric soft-tissue masses.
    C. They may form in bursae.
    D. Tophaceous deposits in tendons may result in tendon rupture.
    E. They classically are intra-articular in location.
A
  1. E,. They classically are intra-articular in location.
    Gout is the clinical condition of symptomatic arthritis due to the deposition of monosodium urate crystals within or around the joints due to excess serum urate levels. There is often accompanying renal disease with formation of uric acid urinary’ calculi. Tophaceous gout presents with an eccentric, asymmetric soft tissue mass(es) around the joint(s) and it is this that gives rise to the typical juxta articular erosions. Intra-articular erosions can also occur, though this is less common. Large overhanging edges of bone often develop, separating the tophi from the erosions.
115
Q
  1. A 9-month-old boy with recurrent urinary’ tract infections and renal pelvis dilatation is referred for a micturating cystourethrogram (MCUG). The patient is currently taking a prophylactic dose of trimethoprim.
    With regard to performing an MCUG in this patient, which of the following instructions would you give his mother?
    A. The patient should continue on his current regime of antibiotics until his physicians advise him to stop.
    B. The patient should stop taking their antibiotics the day before the procedure and restart the medication the day after the procedure.
    C. The patient should take a course of treatment dose trimethoprim, starting on the day of the procedure for 3 subsequent days.
    D. The patient should take a 5-day course of treatment dose trimethoprim, starting on the day before the procedure.
    E. The patient should stop their antibiotics the day before the procedure and take a 3 day course of treatment dose gentamicin.
A
  1. C. The patient should take a course of treatment dose trimethoprim, starting on the day of
    the procedure for 3 subsequent days.
    Ensure that the baby undergoing MCUG receives 4 mg/kg of trimethoprim on the day of the MCUG and for three subsequent days to prevent complication of infection. If a child is on prophylactic 2 mg/kg does of trimethoprim, full dose is required as described above to cover the MCUG.
116
Q
  1. Which of the following is incorrect?
A
  1. D. Choroid plexus papilloma Poorly circumscribed and poorly enhancing tumours Predominant imaging patterns of astrocytoma are mass with a enhancing or non-enhancing cyst
    and an intensely enhancing mural nodule, necrotic mass with a central non-enhancing zone or predominantly solid mass. They are associated with NFL Craniopharyngiomas are relatively benign neoplasms arising in the sellar/suprasellar region, often presenting with endocrine abnormalities. Paediatric craniopharyngiomas typically appear multicystic the solid portions enhancing heterogeneously with characteristic calcifications in paediatric craniopharyngiomas may not be discernible without SWI. Occasionally, they are predominantly solid, typically without calcification.
    PNETs arise from primitive (undifferentiated) brain cells. They are typically heterogeneous and are usually iso- to hyperdense on CT, with calcifications in 70%. On MRI, they may appear iso- to hyperintense to grey matter on FLAIR and T2-weighted images. The solid component usually has avid heterogeneous enhancement with minimal oedema. Necrosis and haemorrhage are common and are seen as restriction on DWI.
    On MR images, choroid plexus papillomas appear as isointense to hypointense intraventricular masses compared with normal brain parenchyma. Flow voids, consistent with flowing blood, are common. CT show intense enhancement. Hydrocephalus is very common.
    Gangliogliomas develop in the temporal lobe and are commonly associated with refractory seizures, partial complex type. Calcification is seen in 30%. Peripherally located gangliogliomas may cause scalloped pressure erosion of the calvaria. At MRI, a well-defined cystic mass with a solid mural nodule is typically seen; however, a solid mass with non specific low to intermediate signal intensity on T1-weighted images and high signal on T2-weighted images is also not uncommon. Enhancement is variable, ranging from non-enhancing to ring like to intense homogeneity. There is usually little associated mass effect or oedema.
117
Q
  1. A chest X-ray is performed on a patient 1 week following a left pneumonectomy for bronchogenic carcinoma. Which of the following would you not normally expect to see?
    A. Loss of volume of the left hemithorax
    B. Mediastinal shift to the left
    C. Tracheal shift to the right
    D. An enlarging left-sided hydrothorax
    E. Elevation of the left hemidiaphragm
A
  1. C. Tracheal shift to the right
    Tracheal shift to the contralateral side from the pneumonectomy is an abnormal finding and along with depression of the ipsilateral hemidiaphragm may represent a bronchopleural fistula, empyema or haemorrhage. The other findings can be normally seen postoperatively following pneumonectomy. They can start to develop within 24 hours following surgery with partial and then complete filling of the thorax. The other signs such as ipsilateral mediastinal shift and elevation of the hemidiaphragm are caused by volume loss. When presented with a chest X-ray with complete white-out and ipsilateral mediastinal shift, the key to identifying pneumonectomy versus complete collapse is the presence of a posterior surgical rib defect.
118
Q
  1. A 74-year old man with painless haematuria and weight-loss with irregular thickening of the bladder wall on US underwent an MRI for staging. MRI revealed focal plaque-like thickening in the bladder wall and areas of thin calcification on plain film. All of the following are recognised findings in squamous cell carcinoma of the bladder, except
    A. Single enhancing bladder mass
    B. Papillary tumour with pure intraluminal growth
    C. Sessile enhancing tumour mass
    D. Calcification related to tumour
    E. Mass in a diverticula
A
  1. B. Papillary tumour with pure intraluminal growth
    Squamous cell carcinoma accounts for <5% of bladder neoplasms; however, in parts of the world where schistosomiasis (bilharziasis) is endemic, it is a major health problem, accounting for over 50% of bladder cancers. Risk factors in non-bilharzial regions include chronic irritation from indwelling catheters, bladder calculi or chronic infection. All of these risk factors may be present in paraplegic patients, putting them at increased risk. Cyclophosphamide, smoking and intravesical BCG has also been implicated in the pathogenesis of squamous carcinoma of the bladder.
    The imaging findings in squamous carcinoma are non-specific. Tumours may appear as a single enhancing bladder mass or as diffuse or focal wall thickening. Intradiverticular squamous tumours are soft-tissue masses, sometimes with surface calcification. In contrast to urothelial carcinoma, squamous carcinoma is sessile rather than papillary, and pure intraluminal growth is not seen. Bladder wall thickening and calcification, from chronic inflammation or infection with Bilharzia, may coexist and complicate the diagnosis.
119
Q
  1. A restrained driver of a car involved in a high-speed road traffic accident is brought to the emergency department. He is hypovolaemic, tachycardic and has a very tender abdomen. After stabilisation, an urgent intravenous contrast-enhanced CT scan is performed which show features of significant mesenteric injury and the patient is immediately taken to theatre.
    Which CT sign is not specific for significant mesenteric injury?
    A. Mesenteric contrast extravasation
    B. Mesenteric fat infiltration
    C. Mesenteric vascular beading
    D. Termination of mesenteric vessels
    E. Bowel ischaemia
A
  1. B. Mesenteric fat infiltration
    Significant mesenteric injury requires urgent vascular surgical treatment and, if missed, can lead to life-threatening bowel ischaemia. The most specific CT feature of significant mesenteric injury is mesenteric contrast extravasation, which is almost 100% sensitive. Other highly specific features are vascular beading (irregularity of mesenteric vessels) and termination of mesenteric vessels, which, when seen, are an indication for vascular surgery. Lack of bowel wall enhancement is highly suggestive of segmental bowel ischaemia secondary to significant vascular injury. Mesenteric fat infiltration is a common but very non-specific feature of abdominal trauma. It may be present in bowel wall laceration, contusion or incomplete bowel wall tear as well as in significant bowel and mesenteric injuries